PREPU WK 1 CHP 12, 65, 70, 68, 63

Lakukan tugas rumah & ujian kamu dengan baik sekarang menggunakan Quizwiz!

A nurse suspects that a client has Huntington disease based on which assessment finding? Correct response: Chorea Explanation: The most prominent clincial features of Huntington disease include chorea, intellectual decline, and often emotional disturbance. As the disease progresses, speech becomes slurred, gait becomes disorganized, and cognitive function is altered with dementia.

Which diagnostic is most commonly used for spinal cord compression? Correct response: Magnetic resonance imaging (MRI) Explanation: MRI is the most commonly used diagnostic tool, detecting epidural spinal cord compression and metastases.

The anatomy instructor is discussing the central nervous system. A student asks where the cerebral cortex is located. What should the anatomy instructor answer?

Correct response: "It is located on the surface of the cerebrum." Explanation: The cerebral cortex is the surface of the cerebrum. It contains motor neurons, which are responsible for movement, and sensory neurons, which receive impulses from peripheral sensory neurons located throughout the body.

Age-related changes in the neurologic system must be carefully assessed. Which of the following changes does the nurse expect to find in some degree depending on the patient's age and medical condition? Select all that apply.

Correct response: Decreased muscle mass Increased sensitivity to heat and cold Reduced papillary responses Explanation: Refer to Table 43-5 in the text.

Question 1 See full question12sReport this Question To avoid the side effects of corticosteroids, which medication classification is used as an alternative to treat inflammatory conditions of the eyes? You Selected: NSAIDs Correct response: NSAIDs Explanation: NSAIDs are used as an alternative in controlling inflammatory eye conditions and postoperatively to reduce inflammation. Miotics are used to cause the pupil to constrict. Mydriatics cause the pupil to dilate. Cycloplegics cause paralysis of the iris sphincter. Reference: Hinkle, J.L., & Cheever, K.H., Brunner & Suddarth's Textbook of Medical-Surgical Nursing, 14th ed., Philadelphia, Wolters Kluwer, 2018, Chapter 63: Assessment and Management of Patients With Eye and Vision Disorders, p. 1888. Chapter 63: Assessment and Management of Patients With Eye and Vision Disorders - Page 1888 Add a Note Question 2 See full question56sReport this Question The nurse is providing care to a client who has been admitted to the hospital for treatment of an infection. The client is visually impaired. Which of the following would be most appropriate for the nurse to do when interacting with the client? You Selected: Face the client when speaking directly to him. Correct response: Face the client when speaking directly to him. Explanation: When interacting with a client with a visual impairment, the nurse should face the client and speak directly to the client using a normal tone of voice. It is not necessary to raise the voice unless the client asks the nurse to do so and it is not necessary to avoid the terms, "see" or "look" when interacting with the client. The nurse should identify himself or herself when approaching the client and before making any physical contact. Reference: Hinkle, J.L., & Cheever, K.H., Brunner & Suddarth's Textbook of Medical-Surgical Nursing, 14th ed., Philadelphia, Wolters Kluwer, 2018, Chapter 63: Assessment and Management of Patients With Eye and Vision Disorders, Chart 63-3, p. 1886. Chapter 63: Assessment and Management of Patients With Eye and Vision Disorders - Page 1886 Add a Note Question 3 See full question1m 24sReport this Question After surgery for removal of cataract, a client is being discharged, and the nurse has completed discharge instruction. Which client statement indicates that the outcome of the teaching plan has been met? You Selected: "I should avoid pulling or pushing any object that weighs more than 15 lbs." Correct response: "I should avoid pulling or pushing any object that weighs more than 15 lbs." Explanation: After cataract surgery, the client needs to avoid lifting, pulling, or pushing any object that weighs more than 15 pounds to prevent putting excessive pressure on the surgical site. Sunglasses should be worn when outdoors during the day because the eye is sensitive to light. Dots, flashing lights, a decrease in vision, pain, and increased redness need to be reported to the physician immediately. The eye patch is worn for 24 hours after surgery, followed by eyeglasses worn during the day and a metal shield worn at night for 1 to 4 weeks. Reference: Hinkle, J.L., & Cheever, K.H., Brunner & Suddarth's Textbook of Medical-Surgical Nursing, 14th ed., Philadelphia, Wolters Kluwer, 2018, Chapter 63: Assessment and Management of Patients With Eye and Vision Disorders, Nursing Management, p. 1893. Chapter 63: Assessment and Management of Patients With Eye and Vision Disorders - Page 1893 Add a Note Question 4 See full question53sReport this Question An aging client is brought to the eye clinic by the son. The son states he has seen his parent holding reading materials at an increasing distance to focus properly. What age-related changes does this indicate? You Selected: Presbyopia Correct response: Presbyopia Explanation: Refractive changes, such as presbyopia, occur in older adults where the lens cannot readily accommodate aging. In such cases, the client is observed holding reading materials at an increasing distance to focus properly. In case of a cataract, the client should report increased glare, decreased vision, and changes in color perception. Macular degeneration affects the central vision. Myopia is the inability to see things at a distance clearly. Reference: Hinkle, J.L., & Cheever, K.H., Brunner & Suddarth's Textbook of Medical-Surgical Nursing, 14th ed., Philadelphia, Wolters Kluwer, 2018, Chapter 63: Assessment and Management of Patients With Eye and Vision Disorders, Gerontologic Considerations, p. 1884. Chapter 63: Assessment and Management of Patients With Eye and Vision Disorders - Page 1884 Add a Note Question 5 See full question1mReport this Question When the patient tells the nurse that his vision is 20/200, and asks what that means, the nurse informs the patient that a person with 20/200 vision: You Selected: Sees an object from 20 feet away that a person with normal vision sees from 200 feet away. Correct response: Sees an object from 20 feet away that a person with normal vision sees from 200 feet away. Explanation: The fraction 20/20 is considered the standard of normal vision. Most people, positioned 20 feet from the eye chart, can see the letters designated as 20/20 from a distance of 20 feet. Reference: Hinkle, J.L., & Cheever, K.H., Brunner & Suddarth's Textbook of Medical-Surgical Nursing, 14th ed., Philadelphia, Wolters Kluwer, 2018, Chapter 63: Assessment and Management of Patients With Eye and Vision Disorders, Vision Impairment and Blindness, p. 1883. Chapter 63: Assessment and Management of Patients With Eye and Vision Disorders - Page 1883 Add a Note Question 6 See full question18sReport this Question Which of the following is the role of the nurse toward a patient who is to undergo eye examinations and tests? You Selected: Ensuring that the patient receives eye care to preserve his or her eye function and prevent further visual loss Correct response: Ensuring that the patient receives eye care to preserve his or her eye function and prevent further visual loss Explanation: Although nurses may not be directly involved in caring for patients who are undergoing eye examinations and tests, it is essential that they ensure that patients receive eye care to preserve their eye function and/or prevent further visual loss. The nurse is not involved in conducting the various tests to determine the status of the eyes and in determining if further action is warranted. Patients who are to undergo eye examinations and tests are not required to modify their diet and exercise regimen. Reference: Hinkle, J.L., & Cheever, K.H., Brunner & Suddarth's Textbook of Medical-Surgical Nursing, 14th ed., Philadelphia, Wolters Kluwer, 2018, Chapter 63: Assessment and Management of Patients With Eye and Vision Disorders, Diagnostic Evaluation, p. 1881. Chapter 63: Assessment and Management of Patients With Eye and Vision Disorders - Page 1881 Add a Note Question 7 See full question21sReport this Question Which of the following occurs when there is deviation from perfect ocular alignment? You Selected: Strabismus Correct response: Strabismus Explanation: Strabismus is a condition in which there is deviation from perfect ocular alignment. Ptosis is a drooping eyelids. Chemosis is edema of the conjunctiva. Nystagmus is an involuntary oscillation of the eyeball. Reference: Hinkle, J.L., & Cheever, K.H., Brunner & Suddarth's Textbook of Medical-Surgical Nursing, 14th ed., Philadelphia, Wolters Kluwer, 2018. Add a Note Question 8 See full question22sReport this Question A nurse is obtaining a history from a new client with glaucoma. The client indicates having read about the diagnosis and understanding that this type of glaucoma is due to the degeneration and obstruction of the trabecular meshwork, whose original function is to absorb the aqueous humor. The loss of absorption will lead to an increased resistance, and thus a chronic, painless buildup of pressure in the eye. Which type of glaucoma has the client described? You Selected: open angle Correct response: open angle Explanation: The client described open-angle glaucoma. This type of glaucoma develops painlessly, and visual changes occur slowly. As the IOP rises, it causes edema of the cornea, atrophy of nerve fibers in the peripheral areas of the retina, and degeneration of the optic nerve. Reference: Hinkle, J.L., & Cheever, K.H., Brunner & Suddarth's Textbook of Medical-Surgical Nursing, 14th ed., Philadelphia, Wolters Kluwer, 2018, Chapter 63: Assessment and Management of Patients With Eye and Vision Disorders, p. 1890. Chapter 63: Assessment and Management of Patients With Eye and Vision Disorders - Page 1890 Add a Note Question 9 See full question18sReport this Question The nurse is caring for a client ordered for multiple eye screening. Following which procedure will the nurse instruct the client on a yellow coloring to the skin and urine as being normal? You Selected: Retinal Angiography Correct response: Retinal Angiography Explanation: The nurse is most correct to instruct the client that his skin and urine may turn yellow following a retinal angiography. Sodium fluorescein is a water-soluble dye that is injected into a vein. The dye then travels to the retinal arteries and capillaries, where pictures are obtained of the vascular supply. The other options do not include a dye injection. Reference: Hinkle, J.L., & Cheever, K.H., Brunner & Suddarth's Textbook of Medical-Surgical Nursing, 14th ed., Philadelphia, Wolters Kluwer, 2018, Chapter 63: Assessment and Management of Patients With Eye and Vision Disorders, Nursing Interventions, p. 1882. Chapter 63: Assessment and Management of Patients With Eye and Vision Disorders - Page 1882 Add a Note Question 10 See full question52sReport this Question A nurse practitioner is assessing a patient who is experiencing changes in her vision. The nurse performs the following steps. Place them in the order in which the nurse would complete them. Use all options. You Selected: Obtain an ocular history Test visual acuity Examine the external eye Perform direct ophthalmoscopy Correct response: Obtain an ocular history Test visual acuity Examine the external eye Perform direct ophthalmoscopy Explanation: When completing an assessment for a patient with visual complaints, the nurse would first obtain an ocular history and then test the patient's visual acuity. Next the nurse would examine the external eye and then perform direct ophthalmoscopy to examine the internal eye.

Question 1 See full question20sReport this Question A 52-year-old comes to the clinic for a follow-up examination after being diagnosed with glaucoma. The client states, "I'm hoping that I don't have to use these drops for very long." Which response by the nurse would be most appropriate? You Selected: "You'll need to use the drops for the rest of your life to control the glaucoma." Correct response: "You'll need to use the drops for the rest of your life to control the glaucoma." Explanation: The client is demonstrating a lack of understanding about the condition and its treatment. The nurse needs to provide additional information to the client that the condition can be controlled but not cured. The statement about lifelong therapy would be most appropriate. Eye medications would most likely be needed for the long term, not just a few months. Surgery may be used in conjunction with medication therapy; however, neither method cures the condition. The goal of therapy is to reduce the intraocular pressure to prevent optic nerve damage. In some clients, medication may be all that is needed. In other cases, additional or combintation treatment with surgery or laser procedures may be necessary. Reference: Hinkle, J.L., & Cheever, K.H., Brunner & Suddarth's Textbook of Medical-Surgical Nursing, 14th ed., Philadelphia, Wolters Kluwer, 2018, Chapter 63: Assessment and Management of Patients With Eye and Vision Disorders, Pharmacologic Therapy, p. 1892. Chapter 63: Assessment and Management of Patients With Eye and Vision Disorders - Page 1892 Add a Note Question 2 See full question20sReport this Question Which client statement would lead the nurse to suspect that the client is experiencing bacterial conjunctivitis? You Selected: "My eyelids were stuck together this morning." Correct response: "My eyelids were stuck together this morning." Explanation: Burning, a sensation of a foreign body, and pain in bright light (photophobia) are signs and symptoms associated with any type of conjunctivitis. The drainage related to bacterial conjunctivitis is usually present in the morning, and the eyes may be difficult to open becacuse of adhesions caused by the exudate. Reference: Hinkle, J.L., & Cheever, K.H., Brunner & Suddarth's Textbook of Medical-Surgical Nursing, 14th ed., Philadelphia, Wolters Kluwer, 2018, Chapter 63: Assessment and Management of Patients With Eye and Vision Disorders, Bacterial Conjunctivitis, pp. 1906-1907. Chapter 63: Assessment and Management of Patients With Eye and Vision Disorders - Page 1906-1907 Add a Note Question 3 See full question1m 4sReport this Question A client comes to the eye clinic for a routine check-up. The client tells the nurse he thinks he is color blind. What screening test does the nurse know will be performed on this client to assess for color blindness? You Selected: Ishihara Correct response: Ishihara Explanation: Color vision is assessed with Ishihara polychromatic plates. The client receives a series of cards on which the pattern of a number is embedded in a circle of colored dots. The numbers are in colors that color-blind persons commonly cannot see. Clients with normal vision readily identify the numbers. The Jaeger and the Rosenbaum test near vision while the Snellen tests far vision. Reference: Hinkle, J.L., & Cheever, K.H., Brunner & Suddarth's Textbook of Medical-Surgical Nursing, 14th ed., Philadelphia, Wolters Kluwer, 2018, Chapter 63: Assessment and Management of Patients With Eye and Vision Disorders, Color Vision Testing, p. 1882. Chapter 63: Assessment and Management of Patients With Eye and Vision Disorders - Page 1882 Add a Note Question 4 See full question26sReport this Question Which of the following is the main refracting surface of the eye? You Selected: Cornea Correct response: Cornea Explanation: The cornea is a transparent, avascular, domelike structure that covers the iris, pupil, and anterior chamber. It is the most anterior portion of the eyeball and is the main refracting surface of the eye. The iris is the colored part of the eye. The pupil is a space that dilates and constricts in response to light. Normal pupils are round and constrict symmetrically when a bright light shines on them. The conjunctiva provides a barrier to the external environment and nourishes the eye. Reference: Hinkle, J.L., & Cheever, K.H., Brunner & Suddarth's Textbook of Medical-Surgical Nursing, 14th ed., Philadelphia, Wolters Kluwer, 2018, Chapter 63: Assessment and Management of Patients With Eye and Vision Disorders, Anatomic and Physiologic Overview, p. 1878. Chapter 63: Assessment and Management of Patients With Eye and Vision Disorders - Page 1878 Add a Note Question 5 See full question28sReport this Question After a fall at home, a client hits their head on the corner of a table. Shortly after the accident, the client arrives at the ED, unable to see out of their left eye. The client tells the nurse that symptoms began with seeing spots or moving particles in the field of vision but that there was no pain in the eye. The client is very upset that the vision will not return. What is the most likely cause of this client's symptoms? You Selected: retinal detachment Correct response: retinal detachment Explanation: A detached retina is associated with a hole or tear in the retina caused by stretching or degenerative changes. Retinal detachment may follow a sudden blow, penetrating injury, or eye surgery. Reference: Hinkle, J.L., & Cheever, K.H., Brunner & Suddarth's Textbook of Medical-Surgical Nursing, 14th ed., Philadelphia, Wolters Kluwer, 2018, Chapter 63: Assessment and Management of Patients With Eye and Vision Disorders, p. 1900. Chapter 63: Assessment and Management of Patients With Eye and Vision Disorders - Page 1900 Add a Note Question 6 See full question1m 2sReport this Question A client is in the ED after being struck in the left eye with a baseball, leaving a large ecchymosis and edema. In client education, the nurse explains to the client the functions of the various structures of the eye. What are functions of the eyelids? Select all that apply. You Selected: impact ocular light spread tears Correct response: impact ocular light spread tears Explanation: The eyelids adjust the amount of light that enters the eye and spread tears over the surface of the eye. The eyelids do not produce tears. The eyelids protect against foreign bodies but do not eliminate them. Reference: Hinkle, J.L., & Cheever, K.H., Brunner & Suddarth's Textbook of Medical-Surgical Nursing, 14th ed., Philadelphia, Wolters Kluwer, 2018, Chapter 63: Assessment and Management of Patients With Eye and Vision Disorders, p. 1877. Chapter 63: Assessment and Management of Patients With Eye and Vision Disorders - Page 1877 Add a Note Question 7 See full question22sReport this Question A client is having a routine eye examination. The procedure being performed is done by using an instrument to indent or flatten the surface of the eye. This is known as ________ and it is routinely done to test for ________. You Selected: tonometry; intraocular pressure Correct response: tonometry; intraocular pressure Explanation: The procedure being performed is known as tonometry and it measures intraocular pressure. Reference: Hinkle, J.L., & Cheever, K.H., Brunner & Suddarth's Textbook of Medical-Surgical Nursing, 14th ed., Philadelphia, Wolters Kluwer, 2018, Chapter 63: Assessment and Management of Patients With Eye and Vision Disorders, p. 1881. Chapter 63: Assessment and Management of Patients With Eye and Vision Disorders - Page 1881 Add a Note Question 8 See full question18sReport this Question The nurse is obtaining a visual history from a client who has noted an increase in glare and changes in color perception. Which assessment would the nurse anticipate to confirm a definitive diagnosis? You Selected: Identification of opacities on the lens Correct response: Identification of opacities on the lens Explanation: The client states an increased glare and changes in color perception, which indicates a cataract. Identification of opacities on the lens confirms that diagnosis. A white circle around the cornea and a yellowish aging spot are also symptoms of aging but with different symptoms. Redness of the sclera indicates irritation. Reference: Hinkle, J.L., & Cheever, K.H., Brunner & Suddarth's Textbook of Medical-Surgical Nursing, 14th ed., Philadelphia, Wolters Kluwer, 2018, Chapter 63: Assessment and Management of Patients With Eye and Vision Disorders, Table 63-1: Age-Related Changes in the Eye, p. 1884. Chapter 63: Assessment and Management of Patients With Eye and Vision Disorders - Page 1884 Add a Note Question 9 See full question16sReport this Question What type of medication would the nurse use in combination with mydriatics to dilate the patient's pupil? You Selected: Cycloplegics Correct response: Cycloplegics Explanation: Mydriasis, or pupil dilation, is the main objective of the administration of mydriatics and cycloplegics (Table 63-3). These two types of medications function differently and are used in combination to achieve the maximal dilation that is needed during surgery and fundus examinations to give the ophthalmologist a better view of the internal eye structures. Reference: Hinkle, J.L., & Cheever, K.H., Brunner & Suddarth's Textbook of Medical-Surgical Nursing, 14th ed., Philadelphia, Wolters Kluwer, 2018, Chapter 63: Assessment and Management of Patients With Eye and Vision Disorders, Mydriatic and Cycloplegic Agents, p. 1887. Chapter 63: Assessment and Management of Patients With Eye and Vision Disorders - Page 1887 Add a Note Question 10 See full question20sReport this Question A patient is to have an angiography done using fluorescein as a contrast agent to determine if the patient has macular edema. What laboratory work should the nurse monitor prior to the angiography? You Selected: BUN and creatinine Correct response: BUN and creatinine Explanation: Angiography is done using fluorescein or indocyanine green as contrast agents. Fluorescein angiography is used to evaluate clinically significant macular edema, document macular capillary nonperfusion, and identify retinal and choroidal neovascularization (growth of abnormal new blood vessels) in age-related macular degeneration. It is an invasive procedure in which fluorescein dye is injected, usually into an antecubital vein. Prior to the angiography, the patient's blood urea nitrogen (BUN) and creatinine should be checked to ensure that the kidneys will excrete the contrast agent

A nurse is performing a neurologic assessment on a client with a stroke and cannot elicit a gag reflex. This deficit is related to cranial nerve (CN) X, the vagus nerve. What will the nurse consider a priority nursing diagnosis?

Correct response: Risk for aspiration Explanation: CN X, the vagus nerve, involves the gag reflex, laryngeal hoarseness, swallowing ability, and symmetrical rise of the uvula and soft palate. An impaired gag reflex indicates a danger for aspiration and subsequent pneumonia. An impaired vagus nerve will not affect balance, skin integrity, or intracranial adaptive capacity.

A nurse is working in a neurologist's office. The physician orders a Romberg test. The nurse should have the client:

Correct response: close his or her eyes and stand erect. Explanation: In the Romberg test, the client stands erect with the feet close together and eyes closed. If the client sways as if to fall, it is considered a positive Romberg test. All of the other options include components of neurologic tests, indicating neurologic deficits and balance.

There are 12 pairs of cranial nerves. Only three are sensory. Select the cranial nerve that is affected with decreased visual fields.

Correct response: Cranial nerve II Explanation: The three sensory cranial nerves are I, II and VIII. Cranial nerve II (optic) is affected with decreased visual fields and acuity

To help assess a client's cerebral function, a nurse should ask:

Correct response: "Have you noticed a change in your memory?" Explanation: To assess cerebral function, the nurse should ask about the client's level of consciousness, orientation, memory, and other aspects of mental status. Questions about muscle strength help evaluate the client's motor system. Questions about coordination help her assess cerebellar function. Questions about eyesight help the nurse evaluate the cranial nerves associated with vision.

The nurse is instructing a community class when a student asks, "How does someone get super strength in an emergency?" The nurse should respond by describing the action of the:

Correct response: sympathetic nervous system. Explanation: The division of the autonomic nervous system called the sympathetic nervous system regulates the expenditure of energy. The neurotransmitters of the sympathetic nervous system are called catecholamines. During an emergency situation or an intensely stressful event, the body adjusts to deliver blood flow and oxygen to the brain, muscles, and lungs that need to react in the situation. The musculoskeletal system benefits from the sympathetic nervous system as the fight-or-flight effects pump blood to the muscles. The parasympathetic nervous system works to conserve body energy not expend it during an emergency. The endocrine system regulates metabolic processes.

The nurse is caring for a client who is being assessed for brain death. Which are cardinal signs of brain death? Select all that apply. Correct response: Absence of brainstem reflexes Apnea Coma Explanation: The three cardinal signs of brain death on clinical examination area coma, the absence of brain stem reflexes, and apnea. Adjunctive tests, such as cerebral blood flow studies, electroencephalography, transcranial Doppler, and brain stem auditory evoked potential, are often used to confirm brain death.

Which are risk factors for spinal cord injury (SCI)? Select all that apply. Correct response: Young age Alcohol use Drug abuse Explanation: The predominant risk factors for SCI include young age, male gender, and alcohol and drug use. The frequency with which these risk factors are associated with SCI emphasizes the importance of primary prevention.

Low levels of the neurotransmitter serotonin lead to which of the following disease processes?

Correct response: Depression Explanation: A decrease of serotonin leads to depression. A decrease in the amount of acetylcholine causes myasthenia gravis. Parkinson's disease is caused by a depletion of dopamine. Decreased levels of GABA may cause seizures.

The provider orders the Romberg test for a patient. The nurse tells the patient that the provider wants to evaluate his equilibrium by assessing which cranial nerve?

Correct response: VIII Explanation: Cranial nerve VIII (acoustic) can be checked to assess equilibrium status.

A nurse is working in an outpatient studies unit administering neurological tests. The client is surprised that paste is used to secure an electroencephalogram and asks how it will be removed from the hair. The nurse is most correct to state that the paste is removed with:

Correct response: standard shampoo. Explanation: Standard shampoo is used to remove the paste, which attached the electrodes to the head. Acetone is not used on the hair. There is no special soap needed. More than warm water is needed to lift and remove the paste.

Question 1 See full question44sReport this Question Which medication classification increases aqueous fluid outflow in the client with glaucoma? You Selected: Cholinergics Correct response: Cholinergics Explanation: Cholinergics increase aqueous fluid outflow by contracting the ciliary muscle, causing miosis, and opening the trabecular meshwork. Beta-blockers decrease aqueous humor production. Alpha-adrenergic agonists decrease aqueous humor production. Carbonic anhydrase inhibitors decrease aqueous humor production. Reference: Hinkle, J.L., & Cheever, K.H., Brunner & Suddarth's Textbook of Medical-Surgical Nursing, 14th ed., Philadelphia, Wolters Kluwer, 2018, Chapter 63: Assessment and Management of Patients With Eye and Vision Disorders, Table 63-5, p. 1893. Chapter 63: Assessment and Management of Patients With Eye and Vision Disorders - Page 1893 Add a Note Question 2 See full question24sReport this Question The nures is assessing a client using an Amsler Grid. The nurse is assessing for which of the following? You Selected: Macular problems Correct response: Macular problems Explanation: The Amsler grid is a test used to assess clients for macular problems. Visual acuity is tested using the Snellen chart. Intraocular pressure is measured using tonometry. Perimetry testing evaluates the field of vision. Reference: Hinkle, J.L., & Cheever, K.H., Brunner & Suddarth's Textbook of Medical-Surgical Nursing, 14th ed., Philadelphia, Wolters Kluwer, 2018, Chapter 63: Assessment and Management of Patients With Eye and Vision Disorders, Amsler Grid, p. 1882. Chapter 63: Assessment and Management of Patients With Eye and Vision Disorders - Page 1882 Add a Note Question 3 See full question24sReport this Question A client has been referred to an ophthalmologist for suspected macular degeneration. The nurse knows to prepare what test for the physician to give the client? You Selected: Amsler grid Correct response: Amsler grid Explanation: Clients with macular problems are tested with an Amsler grid. It is made up of a geometric grid of identical squares with a central fixation point. The examiner instructs the client to stare at the central fixation spot on the grid and report if they see any distortion of the squares. Clients with macular problems may say some of the squares are faded or wavy. An Ishihara polychromatic plate, visual field, or slit lamp test will not diagnose macular degeneration. Reference: Hinkle, J.L., & Cheever, K.H., Brunner & Suddarth's Textbook of Medical-Surgical Nursing, 14th ed., Philadelphia, Wolters Kluwer, 2018, Chapter 63: Assessment and Management of Patients With Eye and Vision Disorders, Amsler Grid, p. 1882. Chapter 63: Assessment and Management of Patients With Eye and Vision Disorders - Page 1882 Add a Note Question 4 See full question33sReport this Question The upper eyelid normally covers the uppermost portion of the iris and is innervated by which cranial nerve? You Selected: III Correct response: III Explanation: The upper lid is innervated by the oculomotor nerve (CN III). Cranial nerve I is the olfactory nerve, cranial nerve II is the optic nerve, and cranial nerve IV is the trochlear nerve. Reference: Hinkle, J.L., & Cheever, K.H., Brunner & Suddarth's Textbook of Medical-Surgical Nursing, 14th ed., Philadelphia, Wolters Kluwer, 2018, Chapter 63: Assessment and Management of Patients With Eye and Vision Disorders, Anatomic and Physiologic Overview, p. 1877. Chapter 63: Assessment and Management of Patients With Eye and Vision Disorders - Page 1877 Add a Note Question 5 See full question7sReport this Question Which of the following eye disorders is caused by an elevated intraocular pressure (IOP)? You Selected: Glaucoma Correct response: Glaucoma Explanation: In glaucoma, there is an abnormally high IOP. Cataracts occur when there is a clouding of the lens. Hyperopia is farsightedness. Myopia is nearsightedness. Reference: Hinkle, J.L., & Cheever, K.H., Brunner & Suddarth's Textbook of Medical-Surgical Nursing, 14th ed., Philadelphia, Wolters Kluwer, 2018, Chapter 63: Assessment and Management of Patients With Eye and Vision Disorders, Glaucoma, p. 1890. Chapter 63: Assessment and Management of Patients With Eye and Vision Disorders - Page 1890 Add a Note Question 6 See full question17sReport this Question Which of the following would be an inaccurate clinical manifestation of a retinal detachment? You Selected: Pain Correct response: Pain Explanation: Patient may report the sensation of a shade or curtain coming across the vision of one eye, cobwebs, bright flashing lights, or the sudden onset of a great number of floaters. Patients do no complain of pain. Reference: Hinkle, J.L., & Cheever, K.H., Brunner & Suddarth's Textbook of Medical-Surgical Nursing, 14th ed., Philadelphia, Wolters Kluwer, 2018, Chapter 63: Assessment and Management of Patients With Eye and Vision Disorders, Clinical Manifestations, p. 1900. Chapter 63: Assessment and Management of Patients With Eye and Vision Disorders - Page 1900 Add a Note Question 7 See full question19sReport this Question A client has noticed recently having clearer vision at a distance than up close. What is the term used to describe this client's visual condition? You Selected: hyperopia Correct response: hyperopia Explanation: Hyperopia is farsightedness. People who are hyperopic see objects that are far away better than objects that are close. Reference: Hinkle, J.L., & Cheever, K.H., Brunner & Suddarth's Textbook of Medical-Surgical Nursing, 14th ed., Philadelphia, Wolters Kluwer, 2018, Chapter 63: Assessment and Management of Patients With Eye and Vision Disorders, p. 1883. Chapter 63: Assessment and Management of Patients With Eye and Vision Disorders - Page 1883 Add a Note Question 8 See full question36sReport this Question The nurse is evaluating the client while taking the color vision test. Which response would the nurse anticipate when caring for a client with normal color vision? You Selected: The nurse would anticipate the client identifying numbers and shapes. Correct response: The nurse would anticipate the client identifying numbers and shapes. Explanation: The nurse is correct to anticipate the client being able to identify numbers and shapes dictated by different color codes. The other options do not test for color vision or indicate an inability to differentiate colors. Reference: Hinkle, J.L., & Cheever, K.H., Brunner & Suddarth's Textbook of Medical-Surgical Nursing, 14th ed., Philadelphia, Wolters Kluwer, 2018, Chapter 63: Assessment and Management of Patients With Eye and Vision Disorders, Color Vision Testing, p. 1881. Chapter 63: Assessment and Management of Patients With Eye and Vision Disorders - Page 1881 Add a Note Question 9 See full question22sReport this Question A nurse is caring for a client scheduled to have angiography of the right eye in 1 hour. What is the highest priority nursing intervention? You Selected: Assess blood urea nitrogen (BUN) and creatinine levels. Correct response: Assess blood urea nitrogen (BUN) and creatinine levels. Explanation: The nurse should assess the BUN and creatinine levels to ensure the client has adequate renal function to excrete the contrast used. Inability to excrete the contrast could lead to complications. Instructing the client to hold his or her head still and educating about the feeling of warmth and metallic taste that might occur are appropriate interventions; assessing renal function remains the highest priority. Assessing for dark yellow to orange urine is appropriate in the postprocedural time frame, so it would not be the highest priority. Reference: Hinkle, J.L., & Cheever, K.H., Brunner & Suddarth's Textbook of Medical-Surgical Nursing, 14th ed., Philadelphia, Wolters Kluwer, 2018, Chapter 63: Assessment and Management of Patients With Eye and Vision Disorders, p. 1882. Chapter 63: Assessment and Management of Patients With Eye and Vision Disorders - Page 1882 Add a Note Question 10 See full question17sReport this Question During a routine physical examination, the nurse practitioner notes that a 72-year-old patient has a significant loss of ability to discriminate words. The patient also states that he has noticed that he has trouble hearing high-frequency sounds. The nurse suspects that the patient has an age-related change in his ears known as: You Selected: Degeneration of the organ of Corti. Correct response: Degeneration of the organ of Corti. Explanation: Degeneration of the organ of Corti causes a decreased ability to discriminate high frequencies or to interpret consonant sounds. Refer to Table 48-2 in the text. Alterations in the vestibulospinal reflex affect balance and gait

Question 1 See full question26sReport this Question A client with an inflammatory ophthalmic disorder has been receiving repeated courses of a corticosteroid ointment, one-half inch in the lower conjunctival sac four times a day as directed. The client reports a headache and blurred vision. The nurse suspects that these symptoms represent: You Selected: increased intraocular pressure (IOP). Correct response: increased intraocular pressure (IOP). Explanation: Headache and blurred vision are symptoms of increased IOP, such as from glaucoma. Ophthalmic corticosteroids may trigger an episode of acute glaucoma in susceptible clients. Although the effects of some drugs may diminish with continued use, this doesn't happen with ophthalmic corticosteroids. Incorrect ointment application doesn't cause headache or blurred vision. Reference: Hinkle, J.L., & Cheever, K.H., Brunner & Suddarth's Textbook of Medical-Surgical Nursing, 14th ed., Philadelphia, Wolters Kluwer, 2018, Chapter 63: Assessment and Management of Patients With Eye and Vision Disorders, Table 63-4, p. 1891. Chapter 63: Assessment and Management of Patients With Eye and Vision Disorders - Page 1891 Add a Note Question 2 See full question28sReport this Question Which feature should a nurse observe during an ophthalmic assessment? You Selected: Appearance of the external eye Correct response: Appearance of the external eye Explanation: During an ophthalmic assessment, the nurse should examine the appearance of the external eye and the pupil responses in the client. A qualified examiner determines internal eye function, visual acuity, and intraocular pressure. Reference: Hinkle, J.L., & Cheever, K.H., Brunner & Suddarth's Textbook of Medical-Surgical Nursing, 14th ed., Philadelphia, Wolters Kluwer, 2018, Chapter 63: Assessment and Management of Patients With Eye and Vision Disorders, p. 1880. Chapter 63: Assessment and Management of Patients With Eye and Vision Disorders - Page 1880 Add a Note Question 3 See full question18sReport this Question Which action should the nurse recommend to a client with blepharitis? You Selected: Keep lid margins clean Correct response: Keep lid margins clean Explanation: Instructions on lid hygiene (to keep the lid margins clean and free of exudates) are given to the client. Treatment of a stye includes warm soaks of the area and incision and drainage. The client is not required to sleep with the face parallel to the floor. Reference: Hinkle, J.L., & Cheever, K.H., Brunner & Suddarth's Textbook of Medical-Surgical Nursing, 14th ed., Philadelphia, Wolters Kluwer, 2018, Chapter 63: Assessment and Management of Patients With Eye and Vision Disorders, Table 63-7, p. 1906. Chapter 63: Assessment and Management of Patients With Eye and Vision Disorders - Page 1906 Add a Note Question 4 See full question24sReport this Question You are the clinic nurse in an ophthalmic clinic. When assessing clients, which client has an abnormal intraocular pressure (IOP)? You Selected: A client with an IOP of 21 mm Hg Correct response: A client with an IOP of 8 mm Hg Explanation: The client with an IOP of 8 mm Hg has a low pressure. The normal IOP is 10 to 21 mm Hg. Reference: Hinkle, J.L., & Cheever, K.H., Brunner & Suddarth's Textbook of Medical-Surgical Nursing, 14th ed., Philadelphia, Wolters Kluwer, 2018, Chapter 63: Assessment and Management of Patients With Eye and Vision Disorders, Anatomic and Physiologic Overview, p. 1878. Chapter 63: Assessment and Management of Patients With Eye and Vision Disorders - Page 1878 Add a Note Question 5 See full question16sReport this Question A legally blind client is in pre-op area prior to an appendectomy. What steps does the nurse take to effectively communicate with this client ? You Selected: Notify the client prior to touching the client. Correct response: Notify the client prior to touching the client. Explanation: The nurse should announce upon arrival the bedside every time because many voices sound similar. The nurse should use the client's name initially so the client knows the nurse is communicating with the client directly. The nurse should speak before touching the client as not to startle the client. The nurse should notify the client when approaching and leaving the bedside each time. Orient the client to their surroundings using verbal descriptions and directions such as left, or right. Reference: Hinkle, J.L., & Cheever, K.H., Brunner & Suddarth's Textbook of Medical-Surgical Nursing, 14th ed., Philadelphia, Wolters Kluwer, 2018, Chapter 63: Assessment and Management of Patients With Eye and Vision Disorders, Chart 63-3, p. 1886. Chapter 63: Assessment and Management of Patients With Eye and Vision Disorders - Page 1886 Add a Note Question 6 See full question19sReport this Question A patient has been diagnosed with hyphema. Which of the following medicationclassifications stabilizes clot formation at the site of hemorrhage? You Selected: Antifibrinolytics Correct response: Antifibrinolytics Explanation: An antifibrinolytic agent, aminocaproic acid (Amicar), stabilizes clot formation at the site of hemorrhage. Corticosteroids, antiplatelets, and diuretics do not have this action. Aspirin is contraindicated. Reference: Hinkle, J.L., & Cheever, K.H., Brunner & Suddarth's Textbook of Medical-Surgical Nursing, 14th ed., Philadelphia, Wolters Kluwer, 2018, Chapter 63: Assessment and Management of Patients With Eye and Vision Disorders, Penetrating Injuries and Contusions of the Eyeball, p. 1904. Chapter 63: Assessment and Management of Patients With Eye and Vision Disorders - Page 1904 Add a Note Question 7 See full question27sReport this Question The nurse is obtaining a history on a client stating the inability to read the newspaper and even seeing detail when looking at an image. Which assessment test would add additional data for a diagnosis? You Selected: Assess vision on the Snellen chart. Correct response: Assess peripheral vision. Explanation: The client states symptoms of the inability to discriminate letters, words, and details of an image, indicating the degeneration of the macula. If the macula is damaged, the client will only have the ability to see movement and gross objects in the peripheral fields. Assessing the peripheral vision will add essential information. The other visual tests are not as important at this time. Reference: Hinkle, J.L., & Cheever, K.H., Brunner & Suddarth's Textbook of Medical-Surgical Nursing, 14th ed., Philadelphia, Wolters Kluwer, 2018, Chapter 63: Assessment and Management of Patients With Eye and Vision Disorders, Perimetry Testing, p. 1883. Chapter 63: Assessment and Management of Patients With Eye and Vision Disorders - Page 1883 Add a Note Question 8 See full question19sReport this Question The nurse at the eye clinic is caring for a patient with suspected glaucoma. What complaint would be significant for a diagnosis of glaucoma? You Selected: The presence of halos around lights Correct response: The presence of halos around lights Explanation: Glaucoma is often called the "silent thief of sight" because most patients are unaware that they have the disease until they have experienced visual changes and vision loss. The patient may not seek health care until he or she experiences blurred vision or "halos" around lights, difficulty focusing, difficulty adjusting eyes in low lighting, loss of peripheral vision, aching or discomfort around the eyes, and headache. Reference: Hinkle, J.L., & Cheever, K.H., Brunner & Suddarth's Textbook of Medical-Surgical Nursing, 14th ed., Philadelphia, Wolters Kluwer, 2018, Chapter 63: Assessment and Management of Patients With Eye and Vision Disorders, Clinical Manifestation, p. 1891. Chapter 63: Assessment and Management of Patients With Eye and Vision Disorders - Page 1891 Add a Note Question 9 See full question17sReport this Question When conducting an eye exam, the nurse practitioner is aware that a diagnostic clinical manifestation of glaucoma is: You Selected: The presence of halos around lights. Correct response: The presence of halos around lights. Explanation: Most patients are unaware that they have glaucoma until they experience visual changes and vision loss. Usually the patient notices blurred vision and the presence of "halos" around lights. Reference: Hinkle, J.L., & Cheever, K.H., Brunner & Suddarth's Textbook of Medical-Surgical Nursing, 14th ed., Philadelphia, Wolters Kluwer, 2018, Chapter 63: Assessment and Management of Patients With Eye and Vision Disorders, Clinical Manifestation, p. 1891. Chapter 63: Assessment and Management of Patients With Eye and Vision Disorders - Page 1891 Add a Note Question 10 See full question17sReport this Question The patient with glaucoma is usually started on the lowest dose of medication. Which of the following is the preferred initial topical medication? You Selected: Beta-blockers Correct response: Beta-blockers Explanation: Because of their efficacy, minimal dosing (can be used once each day), and low cost, beta-blockers are the preferred initial topical medications. Beta-blockers decrease the production of aqueous humor, with a resultant decrease in IOP

Question 1 See full question20sReport this Question A client has just been diagnosed with early glaucoma. During a teaching session, the nurse should: You Selected: demonstrate eyedrop instillation. Correct response: demonstrate eyedrop instillation. Explanation: Eyedrop instillation is a critical component of self-care for a client with glaucoma. After demonstrating eyedrop instillation to the client and family, the nurse should verify their ability to perform this measure properly. An eye patch isn't necessary unless the client has undergone surgery. Visual acuity assessment isn't necessary before discharge. Intraocular lenses aren't implanted in clients with glaucoma. Reference: Hinkle, J.L., & Cheever, K.H., Brunner & Suddarth's Textbook of Medical-Surgical Nursing, 14th ed., Philadelphia, Wolters Kluwer, 2018, Chapter 63: Assessment and Management of Patients With Eye and Vision Disorders, Pharmacologic Therapy, p. 1892. Chapter 63: Assessment and Management of Patients With Eye and Vision Disorders - Page 1892 Add a Note Question 2 See full question20sReport this Question A client accidentally splashes chemicals into one eye. The nurse knows that eye irrigation with plain tap water should begin immediately and continue for 15 to 20 minutes. What is the primary purpose of this first aid treatment? You Selected: To prevent vision loss Correct response: To prevent vision loss Explanation: Prolonged eye irrigation after a chemical burn is the most effective way to prevent formation of permanent scar tissue and thus help prevent vision loss. After a potentially serious eye injury, the victim should always seek medical care. Eye irrigation isn't considered a stopgap measure. Reference: Hinkle, J.L., & Cheever, K.H., Brunner & Suddarth's Textbook of Medical-Surgical Nursing, 14th ed., Philadelphia, Wolters Kluwer, 2018, Chapter 63: Assessment and Management of Patients With Eye and Vision Disorders, Ocular Trauma, p. 1904. Chapter 63: Assessment and Management of Patients With Eye and Vision Disorders - Page 1904 Add a Note Question 3 See full question18sReport this Question The nurse is assessing an older client's vision. The nurse integrates knowledge of which of the following during the assessment? You Selected: The power of the lens to accommodate will be decreased. Correct response: The power of the lens to accommodate will be decreased. Explanation: In the older adult, the accommodative power of the lens decreases, resulting in the need to hold reading materials at increasing distances to focus. Orbital fat and skin elasticity decrease. The depth of the eyeball does not change with age. Reference: Hinkle, J.L., & Cheever, K.H., Brunner & Suddarth's Textbook of Medical-Surgical Nursing, 14th ed., Philadelphia, Wolters Kluwer, 2018, Chapter 63: Assessment and Management of Patients With Eye and Vision Disorders, Gerontologic Considerations, p. 1884. Chapter 63: Assessment and Management of Patients With Eye and Vision Disorders - Page 1884 Add a Note Question 4 See full question24sReport this Question The nurse is reviewing the medical record of a client with glaucoma. Which of the following would alert the nurse to suspect that the client was at increased risk for this disorder? You Selected: Prolonged use of corticosteroids Correct response: Prolonged use of corticosteroids Explanation: Risk factors associated with glaucoma include prolonged use of topical or systemic corticosteroids, older age, myopia, and a history of cardiovascular disease. Reference: Hinkle, J.L., & Cheever, K.H., Brunner & Suddarth's Textbook of Medical-Surgical Nursing, 14th ed., Philadelphia, Wolters Kluwer, 2018, Chapter 63: Assessment and Management of Patients With Eye and Vision Disorders, Corticosteroids and Nonsteroidal Anti-Inflammatory Drugs, p. 1888. Chapter 63: Assessment and Management of Patients With Eye and Vision Disorders - Page 1888 Add a Note Question 5 See full question18sReport this Question A colleague has been splashed in the eye with cleaning solution. Which of the following would be the priority? You Selected: Irrigating the eye immediately with tap water Correct response: Irrigating the eye immediately with tap water Explanation: With any ocular burn, the priority is to irrigate the eye with tap water immediately. While or after this is done, information about the substance can be obtained. A local anesthetic is instilled, particulate matter is removed, and irrigation continues until the pH normalizes. Then antibiotics are instilled and the eye is patched. Reference: Hinkle, J.L., & Cheever, K.H., Brunner & Suddarth's Textbook of Medical-Surgical Nursing, 14th ed., Philadelphia, Wolters Kluwer, 2018, Chapter 63: Assessment and Management of Patients With Eye and Vision Disorders, Splash Injuries, p. 1904. Chapter 63: Assessment and Management of Patients With Eye and Vision Disorders - Page 1904 Add a Note Question 6 See full question20sReport this Question A young client is being seen by a pediatric ophthalmologist due to a recent skateboarding accident that resulted in trauma to the right cornea, and is now at risk of developing an infection. Which nursing intervention would be contraindicated for a client at risk for infection? You Selected: To ensure correct application of antibiotic ointment, gently drag tip of tube along lower lid while squeezing ointment on to lid. Correct response: To ensure correct application of antibiotic ointment, gently drag tip of tube along lower lid while squeezing ointment on to lid. Explanation: Avoid contaminating the medication dropper or tube by holding the tip above the eye and adjacent tissue. Using a separate container of ophthalmic medication for each client prevents cross-contamination. Maintaining asepsis prevents the introduction and transmission of infection. Handwashing prevents infection. Reference: Hinkle, J.L., & Cheever, K.H., Brunner & Suddarth's Textbook of Medical-Surgical Nursing, 14th ed., Philadelphia, Wolters Kluwer, 2018, Chapter 63: Assessment and Management of Patients With Eye and Vision Disorders, 63-4, p. 1889. Chapter 63: Assessment and Management of Patients With Eye and Vision Disorders - Page 1889 Add a Note Question 7 See full question16sReport this Question The nurse is assisting the eye surgeon in completing an examination of the eye. Which piece of equipment would the nurse provide to the physician to examine the optic disc under magnification? You Selected: Ophthalmoscope Correct response: Ophthalmoscope Explanation: The nurse is correct to provide an ophthalmoscope to the surgeon for examination of theoptic disc. A retinoscope is used to determine errors in refraction. A tonometer measures intraocular pressure. An Amsler grid tests for problems with the macula. Reference: Hinkle, J.L., & Cheever, K.H., Brunner & Suddarth's Textbook of Medical-Surgical Nursing, 14th ed., Philadelphia, Wolters Kluwer, 2018, Chapter 63: Assessment and Management of Patients With Eye and Vision Disorders, Direct Ophthalmoscopy, p. 1881. Chapter 63: Assessment and Management of Patients With Eye and Vision Disorders - Page 1881 Add a Note Question 8 See full question33sReport this Question A nurse conducted a history and physical for a newly admitted patient who states, "My arms are too short. I have to hold my book at a distance to read." The nurse knows that the patient is most likely experiencing: You Selected: Loss of accommodative power in the lens. Correct response: Loss of accommodative power in the lens. Explanation: Presbyopia is a refractive change that occurs with age. The lens of the eye loses accommodative power. Opacity in the lens indicates a cataract. Reference: Hinkle, J.L., & Cheever, K.H., Brunner & Suddarth's Textbook of Medical-Surgical Nursing, 14th ed., Philadelphia, Wolters Kluwer, 2018, Chapter 63: Assessment and Management of Patients With Eye and Vision Disorders, Gerontologic Considerations, p. 1884. Chapter 63: Assessment and Management of Patients With Eye and Vision Disorders - Page 1884 Add a Note Question 9 See full question17sReport this Question The ophthalmologist tells a patient that he has increased intraocular pressure (IOP). The nurse understands that increased pressure resulting from optic nerve damage is indicated by a reading of: You Selected: >21 mm Hg. Correct response: >21 mm Hg. Explanation: Normal intraocular pressure is 10 to 21 mm Hg. Readings of more than 21 are indicative of increased pressure and probably optic nerve damage. Reference: Hinkle, J.L., & Cheever, K.H., Brunner & Suddarth's Textbook of Medical-Surgical Nursing, 14th ed., Philadelphia, Wolters Kluwer, 2018, Chapter 63: Assessment and Management of Patients With Eye and Vision Disorders, Physiology, p. 1891. Chapter 63: Assessment and Management of Patients With Eye and Vision Disorders - Page 1891 Add a Note Question 10 See full question17sReport this Question There are four major types of ophthalmic procedures to complete a glaucoma examination. If the health care provider wants to inspect the optic nerve, the nurse would prepare the patient for: You Selected: Ophthalmoscopy. Correct response: Ophthalmoscopy. Explanation: Four major types of examinations are used in glaucoma evaluation, diagnosis, and management: tonometry to measure the IOP, ophthalmoscopy to inspect the optic nerve, gonioscopy to examine the filtration angle of the anterior chamber, and perimetry to assess the visual fields

Question 1 See full question9sReport this Question The nurse should monitor for which manifestation in a client who has undergone LASIK? You Selected: Halos and glare Correct response: Halos and glare Explanation: Symptoms of central islands and decentered ablations can occur after LASIK surgery; these include monocular diplopia or ghost images, halos, glare, and decreased visual acuity. These procedures do not cause excessive tearing or result in cataract or stye formation. Reference: Hinkle, J.L., & Cheever, K.H., Brunner & Suddarth's Textbook of Medical-Surgical Nursing, 14th ed., Philadelphia, Wolters Kluwer, 2018, Chapter 63: Assessment and Management of Patients With Eye and Vision Disorders, p. 1899. Chapter 63: Assessment and Management of Patients With Eye and Vision Disorders - Page 1899 Add a Note Question 2 See full question44sReport this Question What would the nurse correctly identify as the structure that responds to light through constriction and dilation. You Selected: Your selection and the correct area, market by the green box. Explanation: The pupil is the structure that responds to light by constricting and dilating. The lens lies directly behind the pupil and allows for focusing. Reference: Hinkle, J.L., & Cheever, K.H., Brunner & Suddarth's Textbook of Medical-Surgical Nursing, 14th ed., Philadelphia, Wolters Kluwer, 2018, Chapter 63: Assessment and Management of Patients With Eye and Vision Disorders, p. 1877. Chapter 63: Assessment and Management of Patients With Eye and Vision Disorders - Page 1877 Add a Note Question 3 See full question1m 23sReport this Question A client suffered trauma to the sclera and is being treated for a subsequent infection. During client education, the nurse indicates where the sclera is attached. Which structure would not be included? You Selected: eyelids Correct response: eyelids Explanation: The sclera does not attach to the eyelids. The sclera protects structures in the eye, and connects directly to the cornea, anterior chamber, iris, and pupil. Reference: Hinkle, J.L., & Cheever, K.H., Brunner & Suddarth's Textbook of Medical-Surgical Nursing, 14th ed., Philadelphia, Wolters Kluwer, 2018, Chapter 63: Assessment and Management of Patients With Eye and Vision Disorders, p. 1878. Chapter 63: Assessment and Management of Patients With Eye and Vision Disorders - Page 1878 Add a Note Question 4 See full question1m 26sReport this Question A nurse is performing an eye examination. Which question would not be included in the examination? You Selected: "Are you able to raise both eyebrows?" Correct response: "Are you able to raise both eyebrows?" Explanation: Asking to raise both eyebrows is a test for cranial nerve VII, the facial nerve, and would not be included in an eye assessment. Reference: Hinkle, J.L., & Cheever, K.H., Brunner & Suddarth's Textbook of Medical-Surgical Nursing, 14th ed., Philadelphia, Wolters Kluwer, 2018, Chapter 63: Assessment and Management of Patients With Eye and Vision Disorders, p. 1880. Chapter 63: Assessment and Management of Patients With Eye and Vision Disorders - Page 1880 Add a Note Question 5 See full question25sReport this Question A client with multiple sclerosis is being seen by a neuroophthalmologist for a routine eye exam. The nurse explains to the client that during the examination, the client will be asked to maintain a fixed gaze on a stationary point while an object is moved from a point on the side, where it can't be seen, toward the center. The client will indicate when the object becomes visible The nurse further explains that the test being performed is called a: You Selected: perimetry test Correct response: perimetry test Explanation: A visual field test or perimetry test measures peripheral vision and detects gaps in the visual field. Reference: Hinkle, J.L., & Cheever, K.H., Brunner & Suddarth's Textbook of Medical-Surgical Nursing, 14th ed., Philadelphia, Wolters Kluwer, 2018, Chapter 63: Assessment and Management of Patients With Eye and Vision Disorders, p. 1883. Chapter 63: Assessment and Management of Patients With Eye and Vision Disorders - Page 1883 Add a Note Question 6 See full question8sReport this Question To avoid the side effects of corticosteroids, which medication classification is used as an alternative in treating inflammatory conditions of the eyes? You Selected: Nonsteroidal anti-inflammatory drugs (NSAIDs) Correct response: Nonsteroidal anti-inflammatory drugs (NSAIDs) Explanation: NSAIDs are used as an alternative in controlling inflammatory eye conditions and postoperatively to reduce inflammation. Miotics are used to cause the pupil to constrict. Mydriatics cause the pupil to dilate. Cycloplegics cause paralysis of the iris sphincter. Reference: Hinkle, J.L., & Cheever, K.H., Brunner & Suddarth's Textbook of Medical-Surgical Nursing, 14th ed., Philadelphia, Wolters Kluwer, 2018, Chapter 63: Assessment and Management of Patients With Eye and Vision Disorders, p. 1888. Chapter 63: Assessment and Management of Patients With Eye and Vision Disorders - Page 1888 Add a Note Question 7 See full question20sReport this Question The nurse is demonstrating how to perform punctal occlusion. Which activities does the nurse perform? You Selected: Applies gentle pressure bilaterally on the bridge of the nose to the inner canthus of each eye Correct response: Applies gentle pressure bilaterally on the bridge of the nose to the inner canthus of each eye Explanation: Punctal occlusion is done by applying gentle pressure to the inner canthus of each eye for 1 to 2 minutes immediately after eye drops are instilled. The nurse does not apply pressure to the eyeball when administering medications. The lower eyelid is held down to expose the conjunctival sac. The other action described will not aid in the retention or absorption of medication. Reference: Hinkle, J.L., & Cheever, K.H., Brunner & Suddarth's Textbook of Medical-Surgical Nursing, 14th ed., Philadelphia, Wolters Kluwer, 2018, Chapter 63: Assessment and Management of Patients With Eye and Vision Disorders, Chart 63-4, p. 1889. Chapter 63: Assessment and Management of Patients With Eye and Vision Disorders - Page 1889 Add a Note Question 8 See full question31sReport this Question It is determined that a patient is legally blind and will be unable to drive any longer. Legal blindness refers to a best-corrected visual acuity (BCVA) that does not exceed what reading in the better eye? You Selected: 20/200 Correct response: 20/200 Explanation: Legal blindness is a condition of impaired vision in which a person has best corrected visual acuity that does not exceed 20/200 in the better eye or whose widest visual field diameter is 20 degrees or less (Prevent Blindness America, 2012). Reference: Hinkle, J.L., & Cheever, K.H., Brunner & Suddarth's Textbook of Medical-Surgical Nursing, 14th ed., Philadelphia, Wolters Kluwer, 2018, Chapter 63: Assessment and Management of Patients With Eye and Vision Disorders, Vision Impairment and Blindness, p. 1883. Chapter 63: Assessment and Management of Patients With Eye and Vision Disorders - Page 1883 Add a Note Question 9 See full question18sReport this Question A patient visits a clinic for an eye examination. He describes his visual changes and mentions a specific diagnostic clinical sign of glaucoma. What is that clinical sign? You Selected: The presence of halos around lights Correct response: The presence of halos around lights Explanation: Colored halos around lights is a classic symptom of acute-closure glaucoma. Reference: Hinkle, J.L., & Cheever, K.H., Brunner & Suddarth's Textbook of Medical-Surgical Nursing, 14th ed., Philadelphia, Wolters Kluwer, 2018, Chapter 63: Assessment and Management of Patients With Eye and Vision Disorders, Clinical Manifestation, p. 1891. Chapter 63: Assessment and Management of Patients With Eye and Vision Disorders - Page 1891 Add a Note Question 10 See full question23sReport this Question On ocular examination, the health care provider notes severely elevated IOP, corneal edema, and a pupil that is fixed in a semi-dilated position. The nurse knows that these clinical signs are diagnostic of the type of glaucoma known as: You Selected: Acute angle-closure. Correct response: Acute angle-closure. Explanation: Acute angle-closure glaucoma is characterized by the symptoms listed, as well as by being rapidly progressive and accompanied by pain

Which term refers to muscular hypertonicity in a weak muscle, with increased resistance to stretch? Correct response: Spasticity Explanation: Spasticity is often associated with weakness, increased deep tendon reflexes, and diminished superficial reflexes. Akathisia refers to restlessness, an urgent need to move around, and agitation. Ataxia refers to impaired ability to coordinate movement. Myoclonus refers to spasm of a single muscle or group of muscles.

A client has sustained a traumatic brain injury. Which of the following is the priority nursing diagnosis for this client? Correct response: Ineffective airway clearance related to brain injury Explanation: Maintaining an airway is always the priority. All the other choices are appropriate nursing diagnoses for this client, but the priority is maintinence of the airway.

A clinic nurse assesses a client with diabetes who reports taking naproxen (Aleve) and the herb bilberry for osteoarthtitis. To assess for an adverse reaction between naproxen and bilberry, the nurse asks the client

Correct response: "Do you bleed easily?" Explanation: Naproxen, a nonsteroidal anti-inflammatory drug, with the herb bilberry may enhance a client's risk for bleeding. Diarrhea, constipation, or both are frequent adverse reactions to naproxen but not bilberry. Bilberry may cause hypoglycemia. Naproxen does not. There should not be an increased level of pain as a result of the interaction of naproxen and bilberry.

A nurse assesses a patient who has been diagnosed with having a pituitary adenoma that is pressing on the third ventricle. The nurse looks for the associated sign/symptom. What is that sign/symptom? Correct response: Increased intracranial pressure Explanation: All the choices are signs and symptoms that can occur with an adenoma, depending on whether the pressure is exerted on the hypothalamus, the third ventricle, or the optic nerves, chiasm, or tracts. Increased intracranial pressure occurs when the third ventricle is affected.

The nurse educator knows which statement about pituitary adenomas is true? Correct response: Cushing disease can result from a functioning tumor. Explanation: Endocrine disorders can result from the existence of functioning pituitary adenomas. These tumors cause the production of hormones at the anterior pituitary and there may be an increase in various hormones, including cortisol that is responsible for the development of Cushing disease. Pituitary adenomas are rarely seen in the pediatric population. Most pituitary adenomas are benign tumors. The incidence of pituitary adenoma tumors is higher in women than men

Which of the following is a hallmark of spinal metastases? Correct response: Pain Explanation: Pain is the hallmark of spinal metastases. Nausea, fatigue, and change in LOC may occur, but these are not the hallmark of spinal metastases.

Which client should the nurse assess for degenerative neurologic symptoms? Correct response: The client with Huntington disease. Explanation: Huntington disease is a chronic, progressive, degenerative neurologic hereditary disease of the nervous system that results in progressive involuntary choreiform movement and dementia. Paget disease is a musculoskeletal disorder, characterized by localized rapid bone turnover, most commonly affecting the skull, femur, tibia, pelvic bones, and vertebrae. Osteomyelitis is an infection of the bone. Malignant glioma is the most common type of brain tumor.

A patient with amyotrophic lateral sclerosis (ALS) asks if the nurse has heard of a drug that will prolong the patient's life. The nurse knows that there is a medication that may prolong the life by 3 to 6 months. To which medication is the patient referring? Correct response: Riluzole Explanation: Riluzole, a glutamate antagonist, has been shown to prolong survival for persons with ALS for 3 to 6 months.

A patient is diagnosed with an aggressive, primary malignant brain tumor. The nurse is aware that the glioma: Correct response: Originated within the brain tissue. Explanation: The most aggressive type of malignant brain tumor is a glioma, which originates within the brain tissue.

A nurse is caring for a client who has a history of a cerebral aneurysm. Which diagnostic test does the nurse anticipate to monitor the status of the aneurysm?

Correct response: cerebral angiography Explanation: The nurse would anticipate a cerebral angiography, which detects distortion of the cerebral arteries and veins . A milligram detects abnormalities of the spinal canal. An electroencephalogram records electrical impulses of the brain. An echoencephalography is an ultrasound of the structures of the brain.

The nurse is caring for a patient who was involved in a motor vehicle accident and sustained a head injury. When assessing deep tendon reflexes (DTR), the nurse observes diminished or hypoactive reflexes. How will the nurse document this finding?

Correct response: 1+ Explanation: Diminished or hypoactive DTRs are indicated by a score of 1+, no response by a score of 0, a normal response by a score of 2+, and an increased response by a score of 3+.

Cranial nerve IX is also known as which of the following?

Correct response: Glossopharyngeal Explanation: Cranial nerve IX is the glossopharyngeal nerve. The vagus nerve is cranial nerve X. Cranial nerve XII is the hypoglossal nerve. The spinal accessory is the cranial nerve XI.

When learning about the nervous system, students learn that which nervous system regulates the expenditure of energy?

Correct response: Sympathetic Explanation: Sympathetic Nervous System: This division of the autonomic nervous system regulates the expenditure of energy.

A nurse is caring for a client with lower back pain who is scheduled for myelography using metrizamide (a water-soluble contrast dye). After the test, the nurse should place the client in which position?

Correct response:Head of the bed elevated 45 degrees Explanation: After a myelogram, positioning depends on the dye injected. When a water-soluble dye such as metrizamide is injected, the head of the bed is elevated to a 45-degree angle to slow the upward dispersion of the dye. The prone and supine positions are contraindicated when a water-soluble contrast dye is used. The client should be positioned supine with the head lower than the trunk after an air-contrast study.

The nurse is caring for a client in the clinic who has come in to have an EMG done. How would the nurse prepare the client for this test?

Correct response:Tell the client to expect some discomfort. Explanation: Tell the client to expect some discomfort when undergoing a lumbar puncture, myelogram, EMG, or nerve conduction studies. There is no fluoroscopy used for an EMG. It is not necessary to lie flat after an EMG.

A patient is suspected of having glaucoma. What reading of IOP would demonstrate an increase resulting from optic nerve damage? Correct response: 21 mm Hg or higher Explanation: Intraocular pressure of greater than 21 mm Hg is a sign of primary open-angle glaucoma.

The nurse asks a client to follow the movement of a pencil up, down, right, left, and both ways diagonally. The nurse is assessing which of the following? You Selected: Extraocular muscle function Correct response: Extraocular muscle function Explanation: The nurse is testing the client's extraocular eye muscle function by having the client follow an object through the six cardinal directions of gaze (up, down, right, left, and both diagonals). Pupillary reaction is tested using a penlight. The nurse observes the position of the eyelids for drooping. The nurse asks a client to stare at an object and then each eye is covered and then uncovered quickly while the examiner looks for any shifts in the eye and oscillations in the eyeball. Reference: Hinkle, J.L., & Cheever, K.H., Brunner & Suddarth's Textbook of Medical-Surgical Nursing, 14th ed., Philadelphia, Wolters Kluwer, 2018, Chapter 63: Assessment and Management of Patients With Eye and Vision Disorders, Anatomic and Physiologic Overview, p. 1877. Chapter 63: Assessment and Management of Patients With Eye and Vision Disorders - Page 1877 Add a Note Question 2 See full question40sReport this Question Which client statement would lead the nurse to suspect that the client is experiencing bacterial conjunctivitis? You Selected: "My eyelids were stuck together this morning." Correct response: "My eyelids were stuck together this morning." Explanation: Burning, a sensation of a foreign body, and pain in bright light (photophobia) are signs and symptoms associated with any type of conjunctivitis. The drainage related to bacterial conjunctivitis is usually present in the morning, and the eyes may be difficult to open becacuse of adhesions caused by the exudate. Reference: Hinkle, J.L., & Cheever, K.H., Brunner & Suddarth's Textbook of Medical-Surgical Nursing, 14th ed., Philadelphia, Wolters Kluwer, 2018, Chapter 63: Assessment and Management of Patients With Eye and Vision Disorders, Bacterial Conjunctivitis, pp. 1906-1907. Chapter 63: Assessment and Management of Patients With Eye and Vision Disorders - Page 1906-1907 Add a Note Question 3 See full question34sReport this Question The nurse is giving a visual field examination to a 55-year-old male client. The client asks what this test is for. What would be the nurse's best answer? You Selected: "This test measures peripheral vision and detects gaps in the visual field." Correct response: "This test measures peripheral vision and detects gaps in the visual field." Explanation: A visual field examination or perimetry test measures peripheral vision and detects gaps in the visual field. Reference: Hinkle, J.L., & Cheever, K.H., Brunner & Suddarth's Textbook of Medical-Surgical Nursing, 14th ed., Philadelphia, Wolters Kluwer, 2018, Chapter 63: Assessment and Management of Patients With Eye and Vision Disorders, Perimetry Testing, p. 1883. Chapter 63: Assessment and Management of Patients With Eye and Vision Disorders - Page 1883 Add a Note Question 4 See full question28sReport this Question Which type of glaucoma presents an ocular emergency? You Selected: Acute angle-closure glaucoma Correct response: Acute angle-closure glaucoma Explanation: Acute angle-closure glaucoma results in rapid progressive visual impairment. Normal tension glaucoma is treated with topical medication. Ocular hypertension is treated with topical medication. Chronic open-angle glaucoma is treated initially with topical medications, with oral medications added at a later time. Reference: Hinkle, J.L., & Cheever, K.H., Brunner & Suddarth's Textbook of Medical-Surgical Nursing, 14th ed., Philadelphia, Wolters Kluwer, 2018, Chapter 63: Assessment and Management of Patients With Eye and Vision Disorders, Table 63-4, p. 1891. Chapter 63: Assessment and Management of Patients With Eye and Vision Disorders - Page 1891 Add a Note Question 5 See full question47sReport this Question Which of the following surgical procedures involves taking a piece of silicone plastic or sponge and sewing it onto the sclera at the site of a retinal tear? You Selected: Scleral buckle Correct response: Scleral buckle Explanation: The scleral buckle is a procedure in which a piece of silicone plastic or sponge is sewn onto the sclera at the site of the retinal tear. The buckle holds the retina against the sclera until scarring seals the tear. The other surgeries do not use this type of procedure. Reference: Hinkle, J.L., & Cheever, K.H., Brunner & Suddarth's Textbook of Medical-Surgical Nursing, 14th ed., Philadelphia, Wolters Kluwer, 2018, Chapter 63: Assessment and Management of Patients With Eye and Vision Disorders, Scleral Buckle, p. 1900. Chapter 63: Assessment and Management of Patients With Eye and Vision Disorders - Page 1900 Add a Note Question 6 See full question42sReport this Question Which surgical procedure involves flattening the anterior curvature of the cornea by removing a stromal lamella layer? You Selected: Laser-assisted stromal in situ keratomileusis (LASIK) Correct response: Laser-assisted stromal in situ keratomileusis (LASIK) Explanation: LASIK involves flattening the anterior curvature of the cornea by removing a stromal lamella or layer. PRK is used to treat myopia and hyperopia with or without astigmatism. Keratoconus is a cone-shaped deformity of the cornea. Keratoplasty involves replacing abnormal host tissue with healthy donor (cadaver) corneal tissue. Reference: Hinkle, J.L., & Cheever, K.H., Brunner & Suddarth's Textbook of Medical-Surgical Nursing, 14th ed., Philadelphia, Wolters Kluwer, 2018, Chapter 63: Assessment and Management of Patients With Eye and Vision Disorders, p. 1899. Chapter 63: Assessment and Management of Patients With Eye and Vision Disorders - Page 1899 Add a Note Question 7 See full question46sReport this Question Which is the most common cause of visual loss in people older than 65 years of age? You Selected: Macular degeneration Correct response: Macular degeneration Explanation: Macular degeneration is the most common cause of visual loss in people older than 65 years of age. Reference: Hinkle, J.L., & Cheever, K.H., Brunner & Suddarth's Textbook of Medical-Surgical Nursing, 14th ed., Philadelphia, Wolters Kluwer, 2018, Chapter 63: Assessment and Management of Patients With Eye and Vision Disorders, p. 1901. Chapter 63: Assessment and Management of Patients With Eye and Vision Disorders - Page 1901 Add a Note Question 8 See full question11sReport this Question A patient comes to the clinic with a suspected eye infection. The nurse recognizes that the patient most likely has conjunctivitis, as evidenced by what symptom? You Selected: A mucopurulent ocular discharge Correct response: A mucopurulent ocular discharge Explanation: Bacterial conjunctivitis manifests with an acute onset of redness, burning, and discharge. There is papillary formation, conjunctival irritation, and injection in the fornices. The exudates are variable but are usually present on waking in the morning. The eyes may be difficult to open because of adhesions caused by the exudate. Purulent discharge occurs in severe acute bacterial infections, whereas mucopurulent discharge appears in mild cases. Reference: Hinkle, J.L., & Cheever, K.H., Brunner & Suddarth's Textbook of Medical-Surgical Nursing, 14th ed., Philadelphia, Wolters Kluwer, 2018, Chapter 63: Assessment and Management of Patients With Eye and Vision Disorders, Microbial Conjunctivitis, p. 1907. Chapter 63: Assessment and Management of Patients With Eye and Vision Disorders - Page 1907 Add a Note Question 9 See full question9sReport this Question A major role for nursing in the management of glaucoma is health education. Which of the following is the most important teaching point that the nurse should advise the patient of? You Selected: Adhere to the medication regimen. Correct response: Adhere to the medication regimen. Explanation: All of the teaching points are important but the most important is emphasizing the strict adherence to the medication regimen because glaucoma cannot be cured but its progression can be slowed. Reference: Hinkle, J.L., & Cheever, K.H., Brunner & Suddarth's Textbook of Medical-Surgical Nursing, 14th ed., Philadelphia, Wolters Kluwer, 2018, Chapter 63: Assessment and Management of Patients With Eye and Vision Disorders, Pharmacologic Therapy, p. 1892. Chapter 63: Assessment and Management of Patients With Eye and Vision Disorders - Page 1892 Add a Note Question 10 See full question15sReport this Question A patient has been diagnosed with bacterial conjunctivitis that was sexually transmitted. The nurse informs the patient that the isolated organism is which of the following? You Selected: Chlamydia trachomatis Correct response: Chlamydia trachomatis Explanation: Common organisms isolated are Streptococcus pneumoniae, Haemophilus influenzae, and Staphylococcus aureus. Two sexually transmitted agents associated with conjunctivitis are Chlamydia trachomatis and Neisseria gonorrhoeae.

The nurse working on a neurological unit is mentoring a nursing student. The student asks about a client who has sustained a primary and and secondary brain injury. The nurse correctly tells the student which of the following, related to the primary injury? Correct response: It results from initial damage to the brain from the traumatic event. Explanation: The primary injury results from the initial damage from the traumatic event. The secondary injury results from inadequate delivery of nutrients and oxygen to the cells, usually due to cerebral edema and increased intracranial pressure.

A client with a spinal cord injury is to receive methylprednisolone sodium succinate 100 mg intravenously twice a day. The medication is supplied in vials containing 125 mg per 2 mL. How many mL will constitute the correct dose? Enter the correct number ONLY. Correct response: 1.6 Explanation: (100 mg/125 mg) x 2 mL = 1.6 mL.

Which of the following is the earliest sign of increasing intracranial pressure (ICP)? Correct response: Loss of consciousness Explanation: The earliest sign of increasing ICP is loss of consciousness. Other manifestations of increasing ICP are vomiting, headache, and posturing.

At a certain point, the brain's ability to autoregulate becomes ineffective and decompensation (ischemia and infarction) begins. Which of the following are associated with Cushing's triad? Select all that apply. Correct response: Bradycardia Hypertension Bradypnea Explanation: The bradycardia, hypertension, and bradypnea associated with this deterioration are known as Cushing's triad, a grave sign. At this point, herniation of the brainstem and occlusion of the cerebral blood flow occur if therapeutic intervention is not initiated immediately.

A nurse observes that decerebrate posturing is a comatose client's response to painful stimuli. Decerebrate posturing as a response to pain indicates:

Correct response: dysfunction in the brain stem. Explanation: Decerebrate posturing indicates damage of the upper brain stem. Decorticate posturing indicates cerebral dysfunction. Increased intracranial pressure is a cause of decortication and decerebration. Alterations in sensation or paralysis indicate dysfunction in the spinal column.

After a motor vehicle crash, a client is admitted to the medical-surgical unit with a cervical collar in place. The cervical spinal X-rays haven't been read, so the nurse doesn't know whether the client has a cervical spinal injury. Until such an injury is ruled out, the nurse should restrict this client to which position? Correct response: Flat, except for logrolling as needed Explanation: When caring for the client with a possible cervical spinal injury who's wearing a cervical collar, the nurse must keep the client flat to decrease mobilization and prevent further injury to the spinal column. The client can be logrolled, if necessary, with the cervical collar on.

The nurse working on a neurological unit is mentoring a nursing student who asks about a client who has sustained primary and secondary brain injuries. The nurse correctly tells the student which of the following, related to the secondary injury? Correct response: It results from inadequate delivery of nutrients and oxygen to the cells. Explanation: Secondary injury results from inadequate delivery of nutrients and oxygen to the cells, usually as a result of cerebral edema and increased intracranial pressure. Primary injury results from initial damage related to the traumatic event.

Which of the following is not a manifestation of Cushing's Triad? Correct response: Tachycardia Explanation: Cushing's triad is manifested by bradycardia, hypertension, and bradypnea. Tachycardia is not a component of the triad.

Which activities would the client with a T4 spinal cord injury be able to perform independently? Select all that apply. Correct response: Eating Breathing Transferring to a wheelchair Writing Explanation: Eating, breathing, transferring to a wheelchair, and writing are functional abilities for those with a T4 injury. Ambulation can be performed independently by a client with an injury at T11-S5 injury.

The client has been brought to the emergency department by their caregiver. The caregiver says that she found the client diaphoretic, nauseated, flushed and complaining of a pounding headache when she came on shift. What are these symptoms indicative of? Correct response: Autonomic dysreflexia Explanation: Characteristics of this acute emergency are as follows: Severe hypertension; Slow heart rate; Pounding headache; Nausea; Blurred vision; Flushed skin; Sweating; Goosebumps (erection of pilomotor muscles in the skin); Nasal stuffiness; and Anxiety. The symptoms in the scenario are not symptoms or concussion, spinal shock, or contusion.

A client has a spinal cord injury. The home health nurse is making an initial visit to the client at home and plans on reinforcing teaching on autonomic dysreflexia. What symptom would the nurse stress to the client and his family? Correct response: Sweating Explanation: Characteristics of this acute emergency are as follows: severe hypertension; slow heart rate; pounding headache; nausea; blurred vision; flushed skin; sweating; goosebumps (erection of pilomotor muscles in the skin); nasal stuffiness; and anxiety.

Pressure ulcers may begin within hours of an acute spinal cord injury (SCI) and may cause delay of rehabilitation, adding to the cost of hospitalization. The most effective approach is prevention. Which of the following nursing interventions will most protect the client against pressure ulcers? Correct response: Meticulous cleanliness Explanation: Meticulous cleanliness is the best choice for preventing pressure ulcers. A continuous indwelling catheter is not conducive to preventing pressure ulcers. Pressure-sensitive areas should be kept well lubricated with lotion. The client does not know the best positioning techniques for prevention of skin breakdown. The nurse and client together should decide how to best position the body.

A client has been brought to the emergency department (ED) after a fall off a roof. The client has no cord function below the point of injury. The ED nurse knows what about this client? Correct response: The client is in spinal shock. Explanation: Spinal shock is a loss of sympathetic reflex activity below the level of injury within 30 to 60 minutes of a spinal injury. It is characterized by immediate loss of all cord functions below the point of injury.

A client has been prescribed a fentanyl patch for pain control. The nurse understands that this patch should be replaced every

Correct response: 48-72 hours. Explanation: Fentanyl patches should be replaced every 48-72 hours, depending on patient response. The other time frames are incorrect.

Which of the following neurotransmitters are deficient in myasthenia gravis?

Correct response: Acetylcholine Explanation: A decrease in the amount of acetylcholine causes myasthenia gravis. A decrease of serotonin leads to depression. Parkinson's disease is caused by a depletion of dopamine. Decreased levels of GABA may cause seizures.

The nurse obtains a Snellen eye chart when assessing cranial nerve function. Which cranial nerve is the nurse testing when using the chart?

Correct response: CN II Explanation: The nurse assesses vision and thus the optic nerve (cranial nerve II) by use of a Snellen eye chart.

Which lobe of the brain is responsible for spatial relationships?

Correct response: Parietal Explanation: The parietal lobe is essential to a person's awareness of body position in space, size and shape discrimination, and right-left orientation. The frontal lobe controls information storage or memory and motor function. The temporal lobe contains the auditory receptive area. The occipital lobe is responsible for visual interpretation.

To assess a client's cranial nerve function, a nurse should assess:

Correct response: gag reflex. Explanation: The gag reflex is governed by the glossopharyngeal nerve, one of the cranial nerves. Hand grip and arm drifting are part of motor function assessment. Orientation is an assessment parameter related to a mental status examination.

The nurse is caring for a client with a head injury. The client is experiencing CSF rhinorrhea. Which order should the nurse question? Correct response: Insertion of a nasogastric (NG) tube Explanation: Clients with brain injury are assumed to be catabolic, and nutritional support consultation should be considered as soon as the client is admitted. Parenteral nutrition via a central line or enteral feedings administered via an NG or nasojejunal feeding tube should be considered. If cerebrospinal fluid rhinorrhea occurs, an oral feeding tube should be inserted instead of a nasal tube. Serial studies of blood and urine electrolytes and osmolality are done because head injuries may be accompanied by disorders of sodium regulation. Urine is tested regularly for acetone. An intervention to maintain skin integrity is getting the client out of bed to a chair three times daily.

The nurse is caring for a client following a spinal cord injury who has a halo device in place. The client is preparing for discharge. Which statement by the client indicates the need for further instruction? Correct response: "I can apply powder under the liner to help with sweating." Explanation: Powder is not used inside the vest because it may contribute to the development of pressure ulcers. The areas around the four pin sites of a halo device are cleaned daily and observed for redness, drainage, and pain. The pins are observed for loosening, which may contribute to infection. If one of the pins becomes detached, the head is stabilized in a neutral position by one person while another notifies the neurosurgeon. The skin under the halo vest is inspected for excessive perspiration, redness, and skin blistering, especially on the bony prominences. The vest is opened at the sides to allow the torso to be washed. The liner of the vest should not become wet because dampness can cause skin excoriation. The liner should be changed periodically to promote hygiene and good skin care.

The nurse is seeing a client who is suspected of having a glioblastoma multiforme tumor. The nurse anticipates the client will require which diagnostic test to confirm the client has this form of brain tumor? Correct response:Tissue biopsy Explanation: Glioblastoma multiforme is the most common and aggressive malignant brain tumor. In most cases, a tissue biopsy, which can be obtained at the time of surgical removal, is needed to confirm the diagnosis. A Weber and Rinne test may be useful in assessing asymmetric hearing loss associated with an acoustic neuroma, not glioblastoma multiforme. The diagnosis of an angioma is suggested by the presence of another angioma somewhere in the head or by a bruit (an abnormal sound) that is audible over the skull. Functioning pituitary adenoma can produce one or more hormones, normally by the anterior pituitary. Increase maybe seen in prolactin hormone, growth hormone, adrenocorticotropic hormone, or thyroid-stimulating hormone.

The nurse is providing health education to a client recently diagnosed with a brain tumor. During the appointment, the client states, "I'm really worried that I will have a seizure while I am at work or with my kids. Should I be concerned about this?" How should the nurse respond? Correct response: "There is a risk for seizures in people who have this diagnosis. What have you already discussed with your primary health care provider regarding management of seizures?" Explanation: In this case, the client is verbalizing a valid concern about management of the potential manifestation of the brain tumor. The nurse should engage the client by providing fact-based information about the probability of seizures caused by effects of brain tumors. The nurse should further engage in the discussion by evaluating the client's existing understanding of the seizures related to brain tumors and the associated management of this problem. The open-ended manner in which the nurse has asked the question in the correct answer option allows the client to reveal any knowledge deficits or gaps in understanding of the condition. Telling the client there is a strong chance that he or she will have a seizure is countertherapeutic and would serve to increase the client's anxiety. The nurse's aim should be to reduce the client's anxiety related to the diagnosis. Telling the client that seizures are a genetic neurological condition is out of context in this situation. The client is worried about having a seizure because he or she has a brain tumor. The nurse should address the concern in the correct context. The nurse is incorrect when stating having this discussion is not within the nurse's scope of practice. The client's verbalized concern presents an opportunity for the nurse to evaluate the client's understanding of the treatment and management of the condition. The nurse should refer the client back to the primary health care provider if there are any aspects of the client's health history that are unclear.

The nurse is assessing the pupils of a patient who has had a head injury. What does the nurse recognize as a parasympathetic effect?

Correct response: Constricted pupils Explanation: Constricted pupils are a parasympathetic effect; dilated pupils are a sympathetic effect

Which term describes the fibrous connective tissues that cover the brain and spinal cord?

Correct response: Meninges Explanation: The meninges have three layers: the dura mater, arachnoid mater, and pia mater. The dura mater is the outermost layer of the protective covering of the brain and spinal cord. The arachnoid is the middle membrane, and the pia mater is the innermost membrane of this protective covering.

The spinal cord is composed of 31 pairs of spinal nerves. How many pairs of thoracic nerves are contained within the spinal column?

Correct response: Twelve Explanation: There are twelve pairs of thoracic nerves, five lumbar and sacral nerves, eight cervical, and one coccygeal.

To avoid the side effects of corticosteroids, which medication classification is used as an alternative to treating inflammatory conditions of the eyes? Correct response: Nonsteroidal anti-inflammatory drugs (NSAIDs) Explanation: NSAIDs are used as an alternative in controlling inflammatory eye conditions and postoperatively to reduce inflammation. Miotics are used to cause the pupil to constrict. Mydriatics cause the pupil to dilate. Cycloplegics cause paralysis of the iris sphincter.

The nurse is establishing a visual test using the Snellen chart for a client experiencing visual changes. At which distance should the nurse instruct the client to stand? Correct response: A 20-feet distance Explanation: The nurse is correct in instructing the client to stand at a 20-feet distance from the Snellen chart. Often, the nurse places tape on the floor to denote the correct distance for the client to stand.

Which condition occurs when blood collects between the dura mater and arachnoid membrane? Correct response: Subdural hematoma Explanation: A subdural hematoma is a collection of blood between the dura mater and the brain, a space normally occupied by a thin cushion of fluid. Intracerebral hemorrhage is bleeding in the brain or the cerebral tissue with displacement of surrounding structures. An epidural hematoma is bleeding between the inner skull and the dura, compressing the brain underneath. An extradural hematoma is another name for an epidural hematoma.

A client with a concussion is discharged after the assessment. Which instruction should the nurse give the client's family? Correct response: Look for signs of increased intracranial pressure Explanation: The nurse informs the family to monitor the client closely for signs of increased intracranial pressure if findings are normal and the client does not require hospitalization. The nurse looks for a halo sign to detect any cerebrospinal fluid drainage.

A nurse is completing a neurological assessment and determines that the client has significant visual deficits. A brain tumor is considered. Considering the functions of the lobes of the brain, which area will most likely contain the neurologic deficit?

Correct response: Occipital Explanation: The vision center is located in the occipital lobe. There is little other functioning that may interfere with the visual process in the other lobes of the brain.

The nurse is completing a neurological assessment and uses the whisper test to assess which cranial nerve?

Correct response: Acoustic Explanation: Clinical examination of the acoustic nerve can be done by the whisper test. Having the client say "ah" tests the vagus nerve. Observing for symmetry when the client performs facial movements tests the facial nerve. The olfactory nerve is tested by having the client identify specific odors.

If a client has a lower motor neuron lesion, the nurse would expect to observe which manifestation upon physical assessment?

Correct response: Decreased muscle tone Explanation: A client with a lower motor neuron lesion would be expected to have decreased muscle tone. Those with upper motor neuron lesion would have hyperactive reflexes, no muscle atrophy, and muscle spasticity.

Which cerebral lobe contains the auditory receptive areas?

Correct response: Temporal Explanation: The temporal lobe plays the most dominant role of any area of the cortex in cerebration. The frontal lobe, the largest lobe, controls concentration, abstract thought, information storage or memory, and motor function. The parietal lobe contains the primary sensory cortex, which analyzes sensory information and relays interpretation to the thalamus and other cortical areas. The occipital lobe is responsible for visual interpretation.

A client with cerebral metastasis suddenly experiences a seizure for which phenytoin 10 mg/kg intravenously is ordered as an initial loading dose. The client weighs 132 pounds. How many milligrams of phenytoin should the client receive? Enter the number ONLY. Correct response: 600 Explanation: First, change the client's weight in pounds to kilograms by dividing the weight by 2.2 (2.2 pounds = 1 kg). The client's weight is 60 kg. Next, set up a proportion: 10/1 = x/60; cross multiply and solve for x, which is 600.

The nurse is caring for a client hospitalized after a motor vehicle accident. The client has a comorbidity of Parkinson's disease. Why should the nurse closely monitor the condition and the drug regimen of a client with Parkinson's disease? Correct response: Drugs administered may cause a wide variety of adverse effects. Explanation: Drugs administered for Parkinsonism may cause a wide variety of adverse effects, which requires careful observation of the client. Over time, clients may respond less and less to their standard drug therapy and have more frequent "off episodes" of hypomobility. As a result, the nurse should administer the drugs closely to the schedule. Generally, a single drug called levodopa is administered to clients with Parkinson's disease. It is also not true that drugs may not cause the requisite therapeutic effect or such clients do not adhere to the drug regimen.

The nurse is caring for a patient in the emergency department with a diagnosed epidural hematoma. What procedure will the nurse prepare the patient for? Correct response: Burr holes Explanation: An epidural hematoma is considered an extreme emergency; marked neurologic deficit or even respiratory arrest can occur within minutes. Treatment consists of making openings through the skull (burr holes) to decrease intracranial pressure emergently, remove the clot, and control the bleeding.

The nurse is caring for a patient diagnosed with an acute subdural hematoma following a craniotomy. The nurse is preparing to administer an IV dose of dexamethasone (Decadron). The medication is available in a 20-mL IV bag and ordered to be infused over 15 minutes. At what rate (mL/hr) will the nurse set the infusion pump? Correct response: 80 Explanation: 20/15 × 60 = 80 mL/hr

A patient has been diagnosed with a brain tumor, a glioblastoma multiforme. The nurse met with the family after the diagnosis to help them understand that: Correct response: Surgery can improve survival time but the results are not guaranteed. Explanation: The overall prognosis for this type of aggressive brain tumor is poor but surgery can improve survival time.

The nurse is providing discharge instructions for a client who was admitted to the oncology unit due to dehydration and anorexia after chemotherapy treatment. What information should the nurse provide to the client to promote improve the client's nutritional intake at home? Correct response: Take prescribed pain medication prior to commencing a meal Explanation: The client needs to be clean, comfortable, and free of pain for meals, in an environment that is as attractive as possible. Ensuring adequate pain relief in advance of commencing a meal will make the experience more pleasant and tolerable. Pain is correlated with lack of appetite. Oral hygiene before meals helps to improve appetite. Offensive sights, sounds, and odors are eliminated. Creative strategies may be required to make food more palatable, provide enough fluids, and increase opportunities for socialization during meals.

A patient has recently been diagnosed with an acoustic neuroma. The nurse helps the patient understand that: Correct response: Hearing loss usually occurs. Explanation: An acoustic neuroma is a benign tumor of the eighth cranial nerve. About 50% can be treated with surgery. Hearing loss always occurs. Compression on the fifth cranial nerve can also cause facial paresthesia.

The nurse is providing postoperative care for a client who just underwent surgery to remove a metastatic intramedullary tumor. On postoperative day 3, the client states, "I am really looking forward to going running again, it had become too difficult because of the loss of feeling in my feet." Which should the nurse address in the client's care plan? Correct response: Knowledge deficit Explanation: Clients with extensive neurologic deficits before surgery usually do not make significant functional recovery, even after successful tumor removal. In this case, the client had already developed bilateral sensory loss in the lower extremities indicating the fairly progressed impact of the tumor on the client's functional ability. The client's statement reflects a knowledge deficit and it is a priority to provide information regarding the possibility that lower extremity sensation may not return. Although body image disturbance and anxiety may be identified and addressed. This would occur after the client demonstrates an accurate understanding of loss of functional capabilities as a result of the progressed tumor. Ensuring the client understands the extent of functional loss due to the impact of the tumor is a priority. The client does not demonstrate impaired cognition.

The nurse in the oncology outpatient clinic receives a phone call from a family member of a client who was diagnosed with a metastatic spinal cord tumor. The family member informs the nurse that the client has been reporting increased back pain in the region of the tumor and dizziness. How should the nurse respond? Correct response: Tell the family member to get the client to hospital for emergency assessment Explanation: The client's reported symptoms are indicative of spinal cord compression, a complication of spinal cord tumors that can lead to permanent paralysis and several other irreversible sensory impairments. Signs and symptoms of spinal cord compression warrant an urgent assessment, because it is an emergency. Providing education regarding pain management, sharing information about expected symptoms and encouraging the client to lie in the prone position are all ineffective and unsafe nursing actions, because the presenting complaints warrant emergency assessment and intervention.

The nurse is seeing a client who has just been diagnosed with a meningioma. The client states he is confused because the provider stated, "If you have to be diagnosed with a brain tumor, this is the least harmful." The client asks the nurse for clarification. How should the nurse respond? Correct response: "I am unable to interpret what your provider meant by making that statement; however, it is true that meningiomas are slow growing tumors that are not typically fatal." Explanation: The nurse should inform the client that nurses cannot interpret what another provider meant by the statement. The nurse can provide client education regarding what is known about the type of brain tumor the client has been diagnosed with. It would be incorrect for the nurse to state that the treatment for this type of brain tumor is aggressive. The tumor is slow growing. and sometime treatment is a 'wait-and-see' approach. Thus, surgery, chemotherapy and radiation would not typically be used together or aggressively. By telling the client the provider was trying to be supportive with the statement that was made communicates to the client that the provider was not telling the truth about the nature of the diagnosis. In this case, the nurse is making an assumption and should not try to interpret for the client what the provider said. It would be countertherapeutic and serve to increase the client's anxiety if the nurse stated discussing the details of the client's diagnosis is not within the nurse's scope. The nurse can provide information about the type of brain tumor within the scope of practice.

nurse is assessing a client with Parkinson's disease. Which of the following would the nurse expect to find? Correct response: Slowing of activity Explanation: Clients with Parkinson's disease typically manifest bradykinesia (slowing of all active movement), a propulsive, forward leaning gait, tremors that disappear with active movement, and muscle rigidity.

Which of the following outcomes would be most appropriate to include in the plan of care for a client diagnosed with a muscular dystrophy? Correct response: Client participates in activities of daily living using adaptive devices. Explanation: The muscular dystrophies are a group of incurable muscle disorders characterized by progressive weakening and wasting of the skeletal or voluntary muscles. Nursing care focuses on maintaining the client at his or her optimal level of functioning and enhancing the quality of life. Therefore, the outcome of participating in activities of daily living with adaptive devices would be most appropriate. Medications are not used to treat these disorders; however, they may be necessary if the client develops a complication such as respiratory dysfunction. The disorder is incurable and progressive, not chronic. Diagnostic follow-up would provide little if any information about the course of the disorder.

What is the only known risk factor for brain tumors? Correct response: Ionizing radiation Explanation: Ionizing radiation is the only known risk factor for brain tumors. Head trauma, use of hair dyes, and the use of cellular phones are possible causes that have been investigated.

A client with a malignant glioma is scheduled for surgery. The client demonstrates a need for additional teaching about the surgery when he states which of the following? Correct response: "The surgeon will be able to remove all of the tumor." Explanation: For clients with malignant gliomas, complete removal of the tumor and cure are not possible but the rationale for resection includes relief of increased intracranial pressure, removal of any necrotic tissue, and reduction in the bulk of the tumor, which theorectically leaves behind fewer cells to become resistant to radiation or chemotherapy.

A patient comes to the emergency department with a large scalp laceration after being struck in the head with a glass bottle. After assessment of the patient, what does the nurse do before the physician sutures the wound? Correct response: Irrigates the wound to remove debris Explanation: Scalp wounds are potential portals of entry for organisms that cause intracranial infections. Therefore, the area is irrigated before the laceration is sutured to remove foreign material and to reduce the risk for infection.

A nurse completes the Glasgow Coma Scale on a patient with traumatic brain injury (TBI). Her assessment results in a score of 6, which is interpreted as: Correct response: Severe TBI. Explanation: A score of 13 to 15 is classified as mild TBI, 9 to 12 is moderate TBI, and 3 to 8 is severe TBI. A score of 3 indicates severe impairment of neurologic function, deep coma, brain death, or pharmacologic inhibition of the neurologic response; a score of 8 or less typically indicates an unconscious patient; a score of 15 indicates a fully alert and oriented patient.

Which of the following is a sympathetic nervous system effect?

Correct response: Decreased peristalsis Explanation: Sympathetic effects of the nervous system include decreased peristalsis, increased blood pressure, dilated pupils, and dilated bronchioles.

A client is waiting in a triage area to learn the medical status of family members following a motor vehicle accident. The client is pacing, taking deep breaths, and handwringing. Considering the effects in the body systems, the nurse anticipates that the liver will:

Correct response: convert glycogen to glucose for immediate use. Explanation: When the body is under stress, the sympathetic nervous system is activated readying the body for action. The effect of the body is to mobilize stored glycogen to glucose to provide additional energy for body action.

Which disease includes loss of motor neurons in the anterior horns of the spinal cord and motor nuclei of the lower brain stem? Correct response: Amyotrophic lateral sclerosis Explanation: Amyotrophic lateral sclerosis (ALS) is a disease of unknown cause in which there is a loss of motor neurons in the anterior horns of the spinal cord and the motor nuclei of the lower brain stem. Parkinson disease is a slowly progressing neurologic movement disorder that eventually leads to disability. Alzheimer disease is a chronic, progressive, and degenerative brain disorder that is accompanied by profound effects on memory, cognition and ability for self-care. Huntington disease is a chronic, progressive, hereditary disease of the nervous system that results in progressive involuntary choreiform movement and dementia.

Impaired balance and uncontrolled tremors of Parkinson's disease is correlated with which neurotransmitter? Correct response: Dopamine Explanation: The impaired balance and uncontrolled tremors of Parkinson's disease have been linked with low levels of dopamine. The other neurotransmitters have not been implicated in Parkinson's disease in this manner.

Autonomic dysreflexia can occur with spinal cord injuries above which of the following levels? Correct response: T6 Explanation: Any patient with a lesion above T6 segment is informed that autonomic dysreflexia can occur and that it may occur even years after the initial injury.

Level of consciousness (LOC) can be assessed based on criteria in the Glasgow Coma Scale (GCS). Which of the following indicators are assessed in the GCS? Select all that apply. Correct response: Eye opening Verbal response Motor response Explanation: LOC can be assessed based on the criteria in the GCS, which include eye opening, verbal response, and motor response. The patient's responses are rated on a scale from 3 to 15. Intelligence and muscle strength are not measured in the GCS.

While snowboarding, a fell and sustained a blow to the head, resulting in a loss of consciousness. The client regained consciousness within an hour after arrival at the ED, was admitted for 24-hour observation, and was discharged without neurologic impairment. What would the nurse expect this client's diagnosis to be? Correct response: concussion Explanation: A concussion results from a blow to the head that jars the brain. It usually is a consequence of falling, striking the head against a hard surface such as a windshield, colliding with another person (e.g., between athletes), battering during boxing, or being a victim of violence. The force of the blow causes temporary neurologic impairment but no serious damage to cerebral tissue. There is generally complete recovery within a short time.

The ED nurse is receiving a client handoff report at the beginning of the nursing shift. The departing nurse notes that the client with a head injury shows Battle sign. The incoming nurse expects which to observe clinical manifestation? Correct response: An area of bruising over the mastoid bone Explanation: Fractures of the base of the skull tend to traverse the paranasal sinus of the frontal bone or the middle ear located in the temporal bone Therefore, they frequently produce hemorrhage from the nose, pharynx, or ears, and blood may appear under the conjunctiva. An area of ecchymosis (bruising) may be seen over the mastoid (Battle sign). Basilar skull fractures are suspected when cerebrospinal fluis (CSF) escapes from the ears (CSF otorrhea) and the nose (CSF rhinorrhea). Drainage of CSF is a serious problem because meningeal infection can occur if organisms gain access to the cranial contents via the nose, ear, or sinus through a tear in the dura. A bloodstain surrounded by a yellowish stain on the head dressing is referred to as a halo sign and is highly suggestive of a CSF leak.

Which of the following diagnostic studies provides visualization of cerebral blood vessels? Correct response: Cerebral angiography Explanation: Cerebral angiography provides visualization of cerebral blood vessels and can localize most cerebral trauma. A PET scan measures the brain's activity and is useful in differentiating tumor from scar tissue or radiation necrosis. Cytologic studies of the cerebral spinal fluid (CSF) may be performed to detect malignant cells because central nervous system tumors can shed cells into the CSF. Computer-assisted stereotactic biopsy is being used to diagnose deep-seated brain tumors.

Which of the following is a late symptom of spinal cord compression? Correct response: Paralysis Explanation: Later symptoms include evidence of motor weakness and sensory deficits progressing to paralysis. Early symptoms associated with spinal cord compression include bladder and bowel dysfunction (urinary incontinence or retention; fecal incontinence or constipation).

A patient has difficulty interpreting his awareness of body position in space. Which lobe is most likely to be damaged?

Correct response: Parietal Explanation: The parietal lobe is the primary sensory cortex. It is essential to a person's awareness of his body in space, as well as orientation in space and spatial relations.

In which location are most brain angiomas located? Correct response: Cerebellum Explanation: Brain angiomas occur most often in the cerebellum. Most brain angiomas do not occur in the hypothalamus, thalamus, or brainstem (midbrain, pons, medulla).

The nurse is caring for a client with mid-to-late stage of an inoperable brain tumor. What teaching is important for the nurse to do with this client? Correct response: Explaining hospice care and services Explanation: The nurse explains hospice care and services to clients with brain tumors that no longer are at a stage where they can be cured. Managing muscle weakness and offering family support groups are important, but explaining hospice is the best answer. Optimizing nutrition at this point is not a priority.

A patient arrives to have an MRI done in the outpatient department. What information provided by the patient warrants further assessment to prevent complications related to the MRI?

Correct response: "I am trying to quit smoking and have a patch on." Explanation: Before the patient enters the room where the MRI is to be performed, all metal objects and credit cards (the magnetic field can erase them) must be removed. This includes medication patches that have a metal backing and metallic lead wires; these can cause burns if not removed (Bremner, 2005).

A nurse is preparing a client for lumbar puncture. The client has heard about post-lumbar puncture headaches and asks how to avoid having one. The nurse tells the client that these headches can be avoided by doing which of the following after the procedure?

Correct response: "Remain prone for 2 to 3 hours." Explanation: The headache is caused by cerebral spinal fluid (CSF) leakage at the puncture site. The supply of CSF in the cranium is depleted so that there is not enough to cushion and stabilize the brain. When the client assumes an upright position, tension and stretching of the venous sinuses and pain-sensitive structures occur. The headache may be avoided if the client remains prone for 2 to 3 hours after the procedure. Drinking plenty of fluids will help in replacing the CSF. Hydration is important for replacement of the CSF lost so remaining NPO is not an option unless it is for other reasons, then IV fluid replacement will be important. Ambulating right away will make the possibility of a headache more likely. It is not necessary to remain on bedrest for more than a few hours, unless a headache has occurred; then bedest for overnight is usually sufficient.

Lesions in the temporal lobe may result in which type of agnosia?

Correct response: Auditory Explanation: Lesions in the temporal lobe (lateral and superior portions) may result in auditory agnosia. Lesions in the occipital lobe may result in visual agnosia. Lesions in the parietal lobe may result in tactile agnosia. Lesions in the parietal lobe (posteroinferior regions) may result in relationship and body part agnosia.

A nurse is caring for a 16-year-old adolescent with a head injury resulting from a fight after a high school football game. A physician has intubated the client and written orders to wean him from sedation therapy. A nurse needs further assessment data to determine whether: Correct response: she'll have to apply restraints to prevent the client from dislodging the endotracheal (ET) tube. Explanation: When the client isn't sedated, he may make attempts to remove the ET tube without realizing what he's doing. The nurse needs to obtain information to determine whether it's necessary to request an order for restraints. The nurse doesn't need to obtain additional data to determine if the nutritional protocol will continue to reflect the client's needs because this aspect of care won't change. The client doesn't require additional assessments to continue I.V. administration of medications. I.V. medication clearly needs to continue because the client is intubated. The staff nurse doesn't need to monitor payment status because client sedation shouldn't affect payment status.

A client with a spinal cord injury has full head and neck control when the injury is at which level? Correct response: C5 Explanation: At level C5, the client retains full head and neck control. At C1 the client has little or no sensation or control of the head and neck. At C2 to C3 the client feels head and neck sensation and has some neck control. At C4 the client has good head and neck sensation and motor control.

The nurse is caring for a client immediately following a spinal cord injury (SCI). Which is an acute complication of SCI? Correct response: Spinal shock Explanation: Acute complications of SCI include spinal and neurogenic shock and deep vein thrombosis (DVT). The spinal shock associated with SCI reflects a sudden depression of reflex activity in the spinal cord (areflexia) below the level of injury. Cardiogenic shock is not associated with SCI. Tetraplegia is paralysis of all extremities after a high cervical spine injury. Paraplegia occurs with injuries at the thoracic level. Autonomic dysreflexia is a long-term complication of SCI.

A patient sustained a head trauma in a diving accident and has a cerebral hemorrhage located within the brain. What type of hematoma is this classified as? Correct response: An intracerebral hematoma Explanation: Intracerebral hemorrhage (hematoma) is bleeding within the brain, into the parenchyma of the brain. It is commonly seen in head injuries when force is exerted to the head over a small area (e.g., missile injuries, bullet wounds, stab injuries). A subdural hematoma (SDH) is a collection of blood between the dura and the brain, a space normally occupied by a thin cushion of cerebrospinal fluid. After a head injury, blood may collect in the epidural (extradural) space between the skull and the dura.

A patient has developed autonomic dysreflexia and all measures to identify a trigger have been unsuccessful. What medication can the nurse provide as ordered by the physician to decrease the blood pressure? Correct response: Hydralazine hydrochloride (Apresoline) IV administered slowly Explanation: If measures to identify and address a trigger do not relieve the hypertension and excruciating headache, a ganglionic blocking agent (hydralazine hydrochloride [Apresoline]) is prescribed and administered slowly by the IV route.

Autonomic dysreflexia is an acute emergency that occurs with spinal cord injury as a result of exaggerated autonomic responses to stimuli. Which of the following is the initial nursing intervention to treat this condition? Correct response: Raise the head of the bed and place the patient in a sitting position. Explanation: The head of the bed is raised and the patient is placed immediately in a sitting position to lower blood pressure. Assessment of body systems is done after the emergency has been addressed.

The critical care nurse is giving end-of-shift report on a client she is caring for. The nurse uses the Glasgow Coma Scale (GCS) to assess the level of consciousness (LOC) of a female client and reports to the oncoming nurse that the client has an LOC of 6. What does an LOC score of 6 in a client indicate?

Correct response: Comatose Explanation: The GSC is used to measure the LOC. The scale consists of three parts: eye opening response, best verbal response, and best motor response. A normal response is 15. A score of 7 or less is considered comatose. Therefore, a score of 6 indicates the client is in a state of coma and not in any other state such as stupor or somnolence. The evaluations are recorded on a graphic sheet where connecting lines show an increase or decrease in the LOC.

The client is scheduled for surgery. The nurse is reviewing with the client about postoperative pain management. The client states her goal after receiving treatment is "0." The first action of the nurse is to

Correct response: Educate the client that this goal may not be achievable. Explanation: The client's goal of complete elimination of pain may be unrealistic. The nurse needs to first teach the client about setting an achievable goal. The nurse will plan to use a combination of pharmacologic and nonpharmacologic interventions for pain relief. The nurse may need to notify the surgeon of the client's goal of "0" for pain relief. The nurse does not ensure large doses of opioids are prescribed for the client. Many factors go into the prescription of medication for pain relief, including the client's response to the medication.

A patient is having a lumbar puncture and the physician has removed 20 mL of cerebrospinal fluid. What nursing intervention is a priority after the procedure?

Correct response: Have the patient lie flat for 6 hours. Explanation: Post-lumbar puncture headache may be avoided if a small-gauge needle is used and if the patient remains prone after the procedure. When more than 20 mL of CSF is removed, the patient is positioned supine for 6 hours (Bader & Littlejohns, 2010).

The sympathetic and parasympathetic nervous systems have a direct effect on the circulatory system. Stimulation of the parasympathetic nervous system (PNS) causes which of the following?

Correct response: Heartbeat to decrease Explanation: The parasympathetic nervous system has a constricting effect on the blood vessels in the heart and skeletal muscles; the heartbeat and blood pressure will decrease.

What is the function of cerebrospinal fluid (CSF)?

Correct response: It cushions the brain and spinal cord. Explanation: CSF is produced primarily in the lateral ventricles of the brain. It acts as a shock absorber and cushions the spinal cord and brain against injury caused by sudden or extreme movement. CSF also functions in the removal of waste products from cerebral tissue. CSF doesn't act as an insulator or a barrier and it doesn't produce cerebral neurotransmitters

A 53-year-old man presents to the emergency department with a chief complaint of inability to form words, and numbness and weakness of the right arm and leg. Where would you locate the site of injury?

Correct response: Left frontoparietal region Explanation: The patient is exhibiting signs of expressive aphasia with numbness/tingling and weakness of the right arm and leg. This indicates injury to the expressive speech center (Broca's area), which is located in the inferior portion of the frontal lobe. The motor strip is located in the posterior portion of the frontal lobe. The sensory strip is located in the anterior parietal lobe.

During a routine physical examination to assess a client's deep tendon reflexes, a nurse should make sure to:

Correct response: support the joint where the tendon is being tested. Explanation: The nurse should support the joint where the tendon is being tested to prevent the attached muscle from contracting. The nurse should use the flat, not pointed, end of the reflex hammer when striking the Achilles tendon. (The pointed end is used to strike over small areas, such as the thumb placed over the biceps tendon.) Tapping the tendon slowly and softly wouldn't provoke a deep tendon reflex response. The nurse should hold the reflex hammer loosely, not tightly, between the thumb and fingers so it can swing in an arc.

A client with spinal cord compression from a tumor must undergo diagnostic testing. Which of the following is the most likely procedure for this client? Correct response: Magnetic resonance imaging Explanation: Magnetic resonance imaging is the most commonly used diagnostic procedure. It is the most sensitive diagnostic tool that is particularly helpful in detecting epidural spinal cord compression and vertebral bone metastases.

Which of the following provides the best outcome for most tumor types? Correct response: Surgery Explanation: Surgical intervention provides the best outcome for most tumor types. The objective of surgical management is the removal of part of or the entire tumor without increasing the neurologic deficit. Radiation, chemotherapy, and palliation may be used for the patient with a brain tumor, but it does not provide the best outcome for most tumor types

Which term is used to describe edema of the optic nerve? Correct response: Papilledema Explanation: Papilledema is edema of the optic nerve. Scotoma is a defect in vision in a specific area in one or both eyes. Lymphedema is the chronic swelling of an extremity due to interrupted lymphatic circulation, typically from an axillary dissection. Angioneurotic edema is a condition characterized by urticaria and diffuse swelling of the deeper layers of the skin.

A client has undergone surgery for a spinal cord tumor that was located in the cervical area. The nurse would be especially alert for which of the following? Correct response: Respiratory dysfunction Explanation: When a spinal tumor is located in the cervical area, respiratory compromise may occur from postoperative edema. Hemorrhage would be a concern with any surgery. Bowel incontinence and skin breakdown are possible but not specific to cervical spinal tumors.

A nurse instructs a client to refrain from blinking after administering eye drops based on which rationale? Correct response: Blinking causes the eye drop to be expelled from the conjunctival sac. Explanation: Blinking expels an instilled eye drop from the conjunctival sac, which interferes wtih the efficacy of the medication. Blood-ocular barriers keep foreign substances from entering the eye. The size of the conjunctival sac does change with blinking. It can hold only 50 uL.

A nurse is reviewing the medical records of several patients who have had their intraocular pressure (IOP) measured:Patient A: IOP 12 mm HgPatient B: IOP 15 mm HgPatient C: IOP 21 mm HgPatient D: IOP 24 mm HgWhich patient would the nurse identify as having increased IOP suggesting glaucoma? Correct response: Patient D Explanation: When IOP is in balance, the pressure ranges from 10 to 21 mm Hg. Increased IOP greater than 21 mm Hg suggests glaucoma.

Which anatomic part supplies cerebrospinal fluid to the subarachnoid space and down the spinal cord on the dorsal surface?

Correct response: Fourth ventricle Explanation: Cerebrospinal fluid (CSF), produced in the ventricles, is circulated around both the brain and the spinal cord by the ventricular system. The fourth ventricle supplies CSF to the subarachnoid space and down the spinal cord on the dorsal surface. The third and fourth ventricles connect via the aqueduct of Sylvius. The arachnoid villus is the area in the brain where CSF is absorbed.

Which Glasgow Coma Scale score is indicative of a severe head injury? Correct response: 7 Explanation: A score between 3 and 8 is generally accepted as indicating a severe head injury.

A client in the intensive care unit (ICU) has a traumatic brain injury. The nurse must implement interventions to help control intracranial pressure (ICP). Which of the following are appropriate interventions to help control ICP? Correct response: Keep the client's neck in a neutral position (no flexing). Explanation: To assist in controlling ICP in clients with severe brain injury, the following are recommended: elevate the head of the bed as prescribed (gravity helps drain fluid), maintain head/neck in neutral alignment (no twisting or flexing), give sedation as ordered to prevent agitation, and avoid noxious stimuli (scatter procedures so that client does not become overtired).

A nurse is providing care to a client who has been diagnosed with metastatic brain cancer. When developing the client's plan of care, which outcome would the nurse most likely identify? Correct response: Improved quality of life Explanation: The treatment of metastatic brain cancer is palliative and involves eliminating or reducing serious symptoms. Even when palliation is the goal, distressing signs and symptoms can be relieved, thereby improving the quality of life for both client and family. Removal of all or part of the tumor is the goal of surgery. Radiation therapy attempts to decrease the incidence of recurrence of incompletely resected tumors.

A client is diagnosed with a brain angioma. When teaching the client about the risks associated with this type of brain tumor, the nurse would educate about signs and symptoms associated with which condition? Correct response: Hemorrhagic stroke Explanation: Brain angiomas (masses composed largely of abnormal blood vessels) are found either in the brain or on its surface. Because the walls of the blood vessels in angiomas are thin, affected clients are at risk for hemorrhagic stroke. Pituitary adenomas that produce hormones can lead to endocrine disorders, such as thyroid disorders. In addition, they can exert pressure on the optic nerves and optic chiasm, leading to vision loss. Acoustic neuromas are associated with hearing loss.

A client is admitted with a cervical spine injury sustained during a diving accident. When planning this client's care, the nurse should assign highest priority to which nursing diagnosis? Correct response: Ineffective breathing pattern Explanation: Because a cervical spine injury can cause respiratory distress, the nurse should take immediate action to maintain a patent airway and provide adequate oxygenation. Impaired physical mobility, Disturbed sensory perception (tactile), and Dressing or grooming self-care deficit may be appropriate for a client with a spinal cord injury — particularly during the course of recovery — but they don't take precedence over a diagnosis of Ineffective breathing pattern.

A client has been diagnosed with a concussion and is to be released from the emergency department. The nurse teaches the family or friends who will be caring for the client to contact the physician or return to the ED if the client Correct response: vomits. Explanation: Vomiting is a sign of increasing intracranial pressure and should be reported immediately. In general, the finding of headache in a client with a concussion is an expected abnormal observation. However, a severe headache, weakness of one side of the body, and difficulty in waking the client should be reported or treated immediately.

The nurse is completing a neurologic assessment and uses the whisper test to assess which cranial nerve?

Correct response: Acoustic Explanation: Clinical examination of the acoustic nerve can be done by the whisper test. Having the client say "ah" tests the vagus nerve. Observing for symmetry when the client performs facial movements tests the facial nerve. The olfactory nerve is tested by having the client identify specific odors.

A Glasgow Coma Scale (GCS) score of 7 or less is generally interpreted as Correct response: coma. Explanation: The Glasgow Coma Scale (GCS) is a tool for assessing a clent's response to stimuli. A score of 7 or less is generally interpreted as coma. A GCS score of 10 or less indicates a need for emergency attention. A GCS score of 3 is interpreted as least responsive; a score of 15 is interpreted as most responsive.

The nurse is caring for a client who has sustained a spinal cord injury (SCI) at C5 and has developed a paralytic ileus. The nurse will prepare the client for which of the following procedures? Correct response: Insertion of a nasogastric tube Explanation: Immediately after a SCI, a paralytic ileus usually develops. A nasogastric tube is often required to relieve distention and to prevent vomiting and aspiration. An enema and digital stimulation will not relieve a paralytic ileus. Bowel surgery is not necessary.

The nurse has documented a client diagnosed with a head injury as having a Glasgow Coma Scale (GCS) score of 7. This score is generally interpreted as Correct response: coma. Explanation: The GCS is a tool for assessing a client's response to stimuli. A score of 7 or less is generally interpreted as a coma. The lowest score is 3 (least responsive/deep coma); the highest is 15 (most responsive). A GCS between 3 and 8 is generally accepted as indicating a severe head injury. No category is termed "least" responsive.

The nurse is planning the care of a patient with a TBI in the neurosurgical ICU. In developing the plan of care, what interventions should be a priority? Select all that apply. Correct response: Making nursing assessments Setting priorities for nursing interventions Anticipating needs and complications Initiating rehabilitation Explanation: The nursing interventions for the patient with a head injury are extensive and diverse. They include making nursing assessments, setting priorities for nursing interventions, anticipating needs and complications, and initiating rehabilitation

The nurse is caring for a patient in the emergency department with a diagnosed epidural hematoma. What procedure will the nurse prepare the patient for? Correct response: Burr holes Explanation: An epidural hematoma is considered an extreme emergency; marked neurologic deficit or even respiratory arrest can occur within minutes. Treatment consists of making openings through the skull (burr holes) to decrease intracranial pressure emergently, remove the clot, and control the bleeding.

Which of the following is a clinical manifestation of pupillary changes that indicate increasing ICP? Correct response: Pupils are showing progressive dilation. Explanation: Pupils that show progressive dilation are indicative of increasing intracranial pressure. Refer to Table 45-1 in the text.

When the nurse observes that the patient has extension and external rotation of the arms and wrists, and extension, plantar flexion, and internal rotation of the feet, she records the patient's posturing as which of the following? Correct response: Decerebrate Explanation: Decerebrate posturing is the result of lesions at the midbrain and is more ominous than decorticate posturing. The described posturing results from cerebral trauma and is not normal. The patient has no motor function, is limp, and lacks motor tone with flaccid posturing. In decorticate posturing, the patient has flexion and internal rotation of the arms and wrists and extension, internal rotation, and plantar flexion of the feet.

At a certain point, the brain's ability to autoregulate becomes ineffective and decompensation (ischemia and infarction) begins. Which of the following are associated with Cushing's triad? Select all that apply. Correct response: Bradycardia Hypertension Bradypnea Explanation: The bradycardia, hypertension, and bradypnea associated with this deterioration are known as Cushing's triad, a grave sign. At this point, herniation of the brainstem and occlusion of the cerebral blood flow occur if therapeutic intervention is not initiated immediately.

A preventative approach to pain relief with non-steroidal anti-inflammatory drugs (NSAIDs) means that the medication is given:

Correct response: Before pain is experienced. Explanation: NSAIDs are most effective for preventive pain management when administered on a fixed-schedule (ie, every 3-4 hours) to prevent the pain experience. When combined with an opioid, the medication regimen is highly effective in managing moderate to severe pain.

The nurse is performing an assessment of cranial nerve function and asks the patient to cover one nostril at a time to see if the patient can smell coffee, alcohol, and mint. The patient is unable to smell any of the odors. The nurse is aware that the patient has a dysfunction of which cranial nerve?

Correct response: CN I Explanation: Cranial nerve (CN) I is the olfactory nerve, which allows the sense of smell. Testing of CN I is done by having the patient identify familiar odors with eyes closed, testing each nostril separately. An inability to smell an odor is a significant finding, indicating dysfunction of this nerve.

The physician's office nurse is caring for a client who has a history of a cerebral aneurysm. Which diagnostic test does the nurse anticipate to monitor the status of the aneurysm?

Correct response: Cerebral angiography Explanation: The nurse would anticipate a cerebral angiography, which detects distortion of the cerebral arteries and veins . A myelogram detects abnormalities of the spinal canal. An electroencephalogram records electrical impulses of the brain. An echoencephalography is an ultrasound of the structures of the brain.

Which occurs when reflexes are hyperactive when the foot is abruptly dorsiflexed?

Correct response: Clonus Explanation: Clonus occurs when the foot is abruptly dorsiflexed. It continues to "beat" two or three times before it settles into a position of rest. Sustained clonus always indicates the present of central nervous system disease and requires further evaluation. Ataxia is incoordination of voluntary muscle action. Rigidity is an increase in muscle tone at rest characterized by increased resistance to passive movement. Flaccid posturing is usually the result of lower brain stem dysfunction; the client has no motor function, is limp, and lacks motor tone.

The pre-nursing class is learning about the nervous system in their anatomy class. What part of the nervous system would the students learn is responsible for digesting food and eliminating body waste?

Correct response: Parasympathetic Explanation: The parasympathetic division of the autonomic nervous system works to conserve body energy and is partly responsible for slowing heart rate, digesting food, and eliminating body wastes.

During a neurological assessment examination, the nurse assesses a patient for tactile agnosia. The nurse places a familiar door key in the patient's hand and asks him to identify the object with his eyes closed. The nurse documents his inability to identify the object and notes the affected area of the brain. Which of the following is the most likely affected area of the brain?

Correct response: Parietal lobe Explanation: The parietal lobe analyzes sensor information and relays the interpretation to the cortical area. Failure to identify a familiar object by touch is indicative of parietal lobe dysfunction. Refer to Table 43-7 in the text.

A patient comes to the emergency department with severe pain in the face that was stimulated by brushing the teeth. What cranial nerve does the nurse understand can cause this type of pain?

Correct response: V Explanation: The trigeminal nerve (cranial nerve V) innervates the forehead, cheeks, and jaw, so pain in the face elicited when brushing the teeth would most likely involve this nerve.

A nurse is performing a neurologic assessment on a client. The nurse observes the client's tongue for symmetry, tremors, and strength, and assesses the client's speech. Which cranial nerve is the nurse assessing?

Correct response: XII Explanation: Cranial nerve XII, the hypoglossal nerve, controls tongue movements involved in swallowing and speech. The tongue should be midline, symmetrical, and free from tremors and fasciculations. The nurse tests tongue strength by asking the client to push his tongue against his cheek as the nurse applies resistance. To test the client's speech, the nurse may ask him to repeat the sentence, "Round the rugged rock that ragged rascal ran." The trochlear nerve (IV) is responsible for extraocular movement (inferior medial). The glossopharyngeal nerve (IX) is responsible for swallowing movements and throat sensations. It's also responsible for taste in the posterior third of the tongue. The abducent nerves (VI) are responsible for lateral extraocular movements.

A client was hit in the head with a ball and knocked unconscious. Upon arrival at the emergency department and subsequent diagnostic tests, it was determined that the client suffered a subdural hematoma. The client is becoming increasingly symptomatic. How would the nurse expect this subdural hematoma to be classified? Correct response: acute Explanation: Subdural hematomas are classified as acute, subacute, and chronic according to the rate of neurologic changes. Symptoms progressively worsen in a client with an acute subdural hematoma within the first 24 hours of the head injury.

Which condition occurs when blood collects between the dura mater and arachnoid membrane? Correct response: Subdural hematoma Explanation: A subdural hematoma is a collection of blooding between the dura mater and brain, a space normally occupied by a thin cushion of fluid. Intracerebral hemorrhage is bleeding in the brain or the cerebral tissue with displacement of surrounding structures. An epidural hematoma is bleeding between the inner skull and the dura, compressing the brain underneath. An extradural hematoma is another name for an epidural hematoma.

A 37-year-old mother of three has just been diagnosed with a grade I meningioma. As part of patient education, the nurse tells the patient that: Correct response: Growth is slow and symptoms are caused by compression rather than tissue invasion. Explanation: A meningioma is benign, encapsulated, and slow-growing. Sometimes the patient has no symptoms because of the slow-growing nature of the tumor.

A nurse helps a patient recently diagnosed with a pituitary adenoma understand that: Correct response: Most tumors produce too much of one or more hormones. Explanation: The majority of these tumors are benign. In rare cases, they may be malignant. Functioning tumors produce hormones, frequently in excessive amounts, resulting in conditions such as hyperthyroidism, Cushing's syndrome, and gigantism or acromegaly.

Which of the following is a hallmark of spinal metastases? Correct response: Pain Explanation: Pain is the hallmark of spinal metastases. Nausea, fatigue, and change in LOC may occur, but these are not the hallmark of spinal metastases.

A nurse knows that a patient exhibiting seizurelike movements localized to one side of the body most likely has what type of tumor? Correct response: A motor cortex tumor Explanation: A tumor in the motor cortex of the frontal lobe produces hemiparesis and partial seizures on the opposite side of the body or generalized seizures. A frontal lobe tumor may also produce changes in emotional state and behavior, as well as an apathetic mental attitude. A cerebellar tumor causes dizziness; an ataxic or staggering gait with a tendency to fall toward the side of the lesion; marked muscle incoordination; and nystagmus (involuntary rhythmic eye movements), usually in the horizontal direction. An occipital lobe tumor produces visual manifestations: contralateral homonymous hemianopsia (visual loss in half of the visual field on the opposite side of the tumor) and visual hallucinations.

A nurse is performing a neurologic assessment on the client and notes a positive Romberg test. This test for balance is related to which of the following cranial nerves?

Correct response: VIII Explanation: CN VIII is the acoustic nerve. It has to do with hearing, air and bone conduction, and balance. CN X is the vagus nerve and has to do with the gag reflex, laryngeal hoarseness, swallowing ability, and the symmetrical rise of the uvula and soft palate. CN III is the oculomotor and has to do with pupillary response, conjugate movements, and nystagmus. CN VII is the facial nerve and has to do with symmetry of facial movements and the ability to discriminate between the taste of sugar and salt.

A client with a spinal cord injury says he has difficulty recognizing the symptoms of urinary tract infection (UTI). Which symptom is an early sign of UTI in a client with a spinal cord injury? Correct response: Fever and change in urine clarity Explanation: Fever and change in urine clarity as early signs of UTI in a client with a spinal cord injury. Lower back pain is a late sign. A client with a spinal cord injury may not experience a burning sensation or urinary frequency.

A client who has been severely beaten is admitted to the emergency department. The nurse suspects a basilar skull fracture after assessing: Correct response: raccoon's eyes and Battle sign. Explanation: A basilar skull fracture commonly causes only periorbital ecchymosis (raccoon's eyes) and postmastoid ecchymosis (Battle sign); however, it sometimes also causes otorrhea, rhinorrhea, and loss of cranial nerve I (olfactory nerve) function. Nuchal rigidity and Kernig's sign are associated with meningitis. Motor loss in the legs that exceeds that in the arms suggests central cord syndrome. Pupillary changes are common in skull fractures with associated meningeal artery bleeding and uncal herniation.

The nurse cares for a client with Huntington disease. What intervention is a priority for safe care? Correct response: Protecting the client from falls Explanation: The client with Huntington disease has a risk for injury from falls and skin breakdown. Protecting the client from falls is a priority for safe care. Electrolyte and cholesterol monitoring is not a priority for this condition. Range-of-motion exercises will not protect the client from injuries.

A patient is diagnosed with a spinal cord tumor and has had a course of radiation and chemotherapy. Two months after the completion of the radiation, the patient complains of severe pain in the back. What is pain an indicator of in a patient with a spinal cord tumor? Correct response: Spinal metastasis Explanation: Pain is the hallmark of spinal metastasis. Patients with sensory root involvement may suffer excruciating pain, which requires effective pain management.

The nurse is caring for a patient who has been hospitalized on several occasions for lower abdominal pain related to Crohn's disease. How may this chronic pain be described?

Correct response: Prolonged in duration Explanation: Acute pain differs from chronic pain primarily in its duration. For example, tissue damage as a result of surgery, trauma, or burns produces acute pain, which is expected to have a relatively short duration and resolve with normal healing. Chronic pain is subcategorized as being of cancer or noncancer origin and can be time limited (e.g., may resolve within months) or persist throughout the course of a person's life.

Which nursing diagnosis takes the highest priority for a client with parkinsonian crisis? Correct response: Ineffective airway clearance Explanation: In parkinsonian crisis, dopamine-related symptoms are severely exacerbated, virtually immobilizing the client. A client confined to bed during such a crisis is at risk for aspiration and pneumonia. Also, excessive drooling increases the risk of airway obstruction. Because of these concerns, the nursing diagnosis of Ineffective airway clearance takes the highest priority. Although Imbalanced nutrition: Less than body requirements, Impaired urinary elimination, and Risk for injury are also appropriate nursing diagnoses, they aren't immediately life-threatening.

Which medication classification should be avoided in the treatment of brain tumors? Correct response: Anticoagulants Explanation: Anticoagulants usually are not prescribed because of the risk for central nervous system (CNS) hemorrhage; however, prophylactic therapy with low-molecular-weight heparin is under investigation. Osmotic diuretics, corticosteroids, and anticonvulsants are utilized in the treatment of brain tumors.

Neurological level of spinal cord injury refers to which of the following? Correct response: The lowest level at which sensory and motor function is normal Explanation: "Neurologic level" refers to the lowest level at which sensory and motor functions are normal. It is not the level of spinal cord transection, the best possible level of recovery, or the highest level at which sensory and motor function is normal.

A client is being treated for a lumbar spinal injury that occurred 5 days ago and is currently experiencing the symptoms of spinal shock. Characteristic for this condition, the client is unable to move the lower extremities, is being closely monitored for hypotension and bradycardia, and has impaired temperature control. Which would not be an expected outcome of care? Correct response: client maintains mechanical ventilation with minimal mucus accumulation Explanation: A client with a lumbar spinal injury would not require mechanical ventilation.

The nurse teaches the client with which disorder that the disease is due to decreased levels of dopamine in the basal ganglia of the brain? Correct response: Parkinson disease Explanation: In some patients, Parkinson disease can be controlled; however, it cannot be cured. Multiple sclerosis is a chronic, degenerative, progressive disease of the central nervous system (CNS) characterized by the occurrence of small patches of demyelination in the brain and spinal cord. Huntington disease is a chronic, progressive, hereditary disease of the nervous system that results in progressive involuntary dancelike movements and dementia. Creutzfeldt-Jakob disease is a rare, transmissible, progressive and fatal disease of the CNS characterized by spongiform degeneration of the gray matter of the brain.

A patient is diagnosed with an aggressive, primary malignant brain tumor. The nurse is aware that the glioma: Correct response: Originated within the brain tissue. Explanation: The most aggressive type of malignant brain tumor is a glioma, which originates within the brain tissue.

A client with end-stage dementia is admitted to the orthopedic unit after undergoing internal fixation of the right hip. How should the nurse manage the client's postoperative pain?

Correct response: Administer analgesics around the clock. Explanation: Because assessing pain medication needs in a client with end-stage dementia is difficult, analgesics should be administered around the clock. Clients at this stage of dementia typically can't request oral pain medications when needed. They're also unable to use patient-controlled analgesia devices. Transdermal patches are used to manage chronic pain; not postoperative pain.

A client is diagnosed with a brain tumor. The nurse's assessment reveals that the client has difficulty interpreting visual stimuli. Based on these findings, the nurse suspects injury to which lobe of the brain?

Correct response: Occipital Explanation: The occipital lobe is responsible for interpreting visual stimuli. The frontal lobe influences personality, judgment, abstract reasoning, social behavior, language expression, and movement. The temporal lobe controls hearing, language comprehension, and storage and memory recall (although memory recall is also stored throughout the brain). The parietal lobe interprets and integrates sensations, including pain, temperature, and touch; it also interprets size, shape, distance, and texture.

A client with a T4 level spinal cord injury (SCI) is complaining of a severe headache. The nurse notes profuse diaphoresis of the client's forehead and scalp. Which of the following does the nurse suspect? Correct response: Autonomic dysreflexia Explanation: Autonomic dysreflexia occurs only after spinal shock has resolved. It is characterized by a severe, pounding headache, marked hypertension, diaphoresis, nausea, nasal congestion, and bradycardia. It occurs only with SCIs above T6 and is a hypertensive emergency. It is not related to thrombophlebitis.

At which of the following spinal cord injury levels does the patient have full head and neck control? Correct response: C5 Explanation: At the level of C5, the patient should have full head and neck control, shoulder strength, and elbow flexion. At C4 injury, the patient will have good head and neck sensation and motor control, some shoulder elevation, and diaphragm movement. At C2 to C3, the patient will have head and neck sensation, some neck control, and can be independent of mechanical ventilation for short periods of time.

Prior to starting a peripheral intravenous line on a patient, what intervention can the nurse provide to decrease the pain from the needle puncture?

Correct response: Apply eutectic mixture of local anesthetic cream 30 minutes prior to the procedure. Explanation: The topical route of administration is used for both acute and chronic pain. For example, the nonopioid diclofenac is available in patch and gel formulations for application directly over painful areas. Local anesthetic creams, such as EMLA (eutectic mixture or emulsion of local anesthetics) and L.M.X.4 (lidocaine cream 4%), can be applied directly over the injection site prior to painful needle stick procedures, and the lidocaine patch 5% is often used for well-localized types of neuropathic pain, such as postherpetic neuralgia.

A client has undergone a lumbar puncture as part of a neurological assessment. The client is put under the care of a nurse after the procedure. Which important postprocedure nursing intervention should be performed to ensure the client's maximum comfort?

Correct response: Encourage the client to drink liberal amounts of fluids Explanation: The nurse should encourage the client to take liberal fluids and should inspect the injection site for swelling or hematoma. These measures help restore the volume of cerebrospinal fluid extracted. The client is administered antihistamines before a test only if he or she is allergic to contrast dye and contrast dye will be used. The room of the client who has undergone a lumbar puncture should be kept dark and quiet. The client should be encouraged to rest, because sensory stimulation tends to magnify discomfort.

A client presents to the emergency department stating numbness and tingling occurring down the left leg into the left foot. When documenting the experience, which medical terminology would the nurse be most correct to report? Correct response: Paresthesia Explanation: When a client reports numbness and tingling in an area, he is reporting a paresthesia. The nurse would document the experience as such or place the client's words in parentheses. The nurse would not make a medical diagnosis of sciatic nerve pain or herniation. The symptoms are not consistent with paralysis.

The nurse in the emergency department is caring for a patient brought in by the rescue squad after falling from a second-story window. The nurse assesses ecchymosis over the mastoid and clear fluid from the ears. What type of skull fracture is this indicative of? Correct response: Basilar skull fracture Explanation: A fracture of the base of the skull is referred to as a basilar skull fracture. Fractures of the base of the skull tend to traverse the paranasal sinus of the frontal bone or the middle ear located in the temporal bone. Therefore, they frequently produce hemorrhage from the nose, pharynx, or ears, and blood may appear under the conjunctiva. An area of ecchymosis (bruising) may be seen over the mastoid (Battle's sign). Basilar skull fractures are suspected when CSF escapes from the ears (CSF otorrhea) and the nose (CSF rhinorrhea).

A new ancillary staff member is assisting the nurse with a client diagnosed with Parkinson's disease. The client needs assistance with eating but doesn't require thickened liquids to aid swallowing. Which instruction should the nurse give the ancillary staff member about eating assistance? Correct response: Make sure the client is sitting with the head of bed elevated to 90 degrees. Explanation:Clients with Parkinson's disease are at risk for aspiration; therefore, the nurse should instruct the ancillary staff member to make sure the head of the client's bed is elevated to 90 degrees before assisting the client with eating. A client doesn't always cough when he aspirates. A client with Parkinson's disease needs fluids to maintain fluid balance. Aspiration is a great concern with Parkinson's disease; therefore; the staff should take precautions to prevent this complication.

Which disease is a chronic, progressive, hereditary disease of the nervous system that results in progressive, involuntary dancelike movements and dementia? Correct response: Huntington disease Explanation: Because it is transmitted as an autosomal dominant genetic disorder, each child of a parent with Huntington disease has a 50% risk of inheriting the illness. Multiple sclerosis is a chronic, degenerative, progressive disease of the central nervous system (CNS) characterized by the occurrence of small patches of demyelination in the brain and spinal cord. Parkinson disease is associated with decreased levels of dopamine due to destruction of pigmented neuronal cells in the substantia nigra in the basal ganglia of the brain. Creutzfeldt-Jakob disease is a rare, transmissible, progressive and fatal disease of the CNS characterized by spongiform degeneration of the gray matter of the brain.

A client with an incurable brain tumor is experiencing nausea and vomiting and has little interest in eating. His family states, "We don't know how to help him." Which of the following would be appropriate for the nurse to suggest to help improve the client's nutritional intake? Select all that apply. Correct response: Ensure that the client is free of pain for meals. Plan meals for times when the client is rested. Provide the client with foods that he likes. Explanation: Suggestions to improve nutrition include making sure that the client is comfortable, free of pain, and rested. This may require family members to adjust meal times. Additionally, they should eliminate offensive sights, sounds, and odors. Therefore, placing the client near sites of meal preparation may be too overwhelming. If the client has difficulty with or shows disinterest in usual foods, the family should offer foods that the client prefers, rather than attempting to get the client to eat as previously. If the client shows marked deterioration, then some other form of nutritional support such as a feeding tube or parenteral nutrition may be indicated, but only if this measure is consistent with the client's choices for care.

Which of the following is a brain tumor arising from the supporting structures? Correct response: Meningiomas Explanation: Brain tumors arising from the supporting structures include meningiomas, neuromas, and pituitary adenomas. Intracerebral tumors include astrocytomas, medulloblastoma, and glioblastoma multiforme.

A patient comes to the emergency department with a large scalp laceration after being struck in the head with a glass bottle. After assessment of the patient, what does the nurse do before the physician sutures the wound? Correct response: Irrigates the wound to remove debris Explanation: Scalp wounds are potential portals of entry for organisms that cause intracranial infections. Therefore, the area is irrigated before the laceration is sutured to remove foreign material and to reduce the risk for infection.

A client is admitted to the hospital after sustaining a closed head injury in a skiing accident. The physician ordered neurologic assessments to be performed every 2 hours. The client's neurologic assessments have been unchanged since admission, and the client is complaining of a headache. Which intervention by the nurse is best? Correct response: Assess the client's neurologic status for subtle changes, administer acetaminophen, and then reassess the client in 30 minutes. Explanation: Headache is common after a head injury. Therefore, the nurse should administer acetaminophen to try to manage the client's pain without causing sedation. The nurse should then reassess the client in 30 minutes to note the effectiveness of the pain medication. Administering codeine, an opioid, could cause sedation that may mask changes in the client's neurologic status. Although a headache is expected, the client should receive treatment to alleviate pain. The nurse should notify the physician if the client's neurologic status changes or if treatment doesn't relieve the headache.

Which of the following is the earliest sign of increasing intracranial pressure (ICP)? Correct response: Change in level of consciousness (LOC) Explanation: The earliest sign of increasing ICP is a change in LOC. Any changes in LOC should be reported immediately. Seizures, restlessness, and pupil changes may occur, but these are not the earliest signs.

The nurse received the report from a previous shift. One of her clients was reported to have a history of basilar skull fracture with otorrhea. What assessment finding does the nurse anticipate? Correct response: The client has cerebral spinal fluid (CSF) leaking from the ear. Explanation: Otorrhea means leakage of CSF from the ear. The client with a basilar skull fracture can create a pathway from the brain to the middle ear due to a tear in the dura. As a result, the client can have cerebral spinal fluid leak from the ear. The nurse may assess clear fluid in the ear canal. Ecchymosis and periorbital edema can be present as a manifestation of bruising from the head injury. An elevated temperature may occur from the head injury and is monitored closely. The client may have serous drainage from the nose especially immediately following the injury.

Which safety action will the nurse implement for a client receiving oxygen therapy who is undergoing magnetic resonance imaging (MRI)?

Correct response: Ensure that no client care equipment containing metal enters the room where the MRI table is located. Explanation: For client safety the nurse must make sure that no client care equipment that contains metal or metal parts (eg, portable oxygen tanks) enters the room where the MRI is located. The client must be assessed for the presence of medication patches with foil backing (e.g., nicotine patch) that may cause a burn. The magnetic field generated by the unit is so strong that any metal-containing items will be strongly attracted and can literally be pulled away with such great force that they can fly like projectiles toward the magnet.

A client in the emergency department has a suspected neurologic disorder. To assess gait, the nurse asks the client to take a few steps; with each step, the client's feet make a half circle. To document the client's gait, the nurse should use which term?

Correct response: Helicopod Explanation: A helicopod gait is an abnormal gait in which the client's feet make a half circle with each step. An ataxic gait is staggering and unsteady. In a dystrophic gait, the client waddles with the legs far apart. In a steppage gait, the feet and toes rise high off the floor and the heel comes down heavily with each step.

The nurse is caring for a client experiencing autonomic dysreflexia. Which of the following does the nurse recognize as the source of symptoms? Correct response: Sympathetic nervous system Explanation: The nurse recognizes that autonomic dysreflexia is an exaggerated sympathetic nervous system response. Symptoms include severe hypertension, slow heart rate, pounding headache, etc. and can lead to seizures, stroke, and death. The autonomic nervous system regulates "feed and breed" functions. The central and peripheral nervous system is a component of the sympathetic nervous system.

A client has been diagnosed with a concussion and is preparing for discharge from the ED. The nurse teaches the family members who will be caring for the client to contact the physician or return to the ED if the client demonstrates reports which complications? Select all that apply. Correct response: Slurred speech Vomiting Weakness on one side of the body Explanation: Clients are discharged from the hospital or ED once they return to baseline after a concussion. Monitoring includes observing the client for a decrease in level of consciousness (LOC), worsening headache, dizziness, seizures, abnormal pupil response, vomiting, irritability, slurred speech, numbness, or weakness in the arms or legs. In general, the finding of headache in the client with a concussion is an expected abnormal observation. However, a severe headache, weakness of one side of the body, and difficulty waking the client should be reported or treated immediately.

A client is scheduled for standard EEG testing to evaluate a possible seizure disorder. Which nursing intervention should the nurse perform before the procedure?

Correct response: Withhold anticonvulsant medications for 24 to 48 hours before the exam Explanation: Anticonvulsant agents, tranquilizers, stimulants, and depressants should be withheld 24 to 48 hours before an EEG because these medications can alter EEG wave patterns or mask the abnormal wave patterns of seizure disorders. To increase the chances of recording seizure activity, it is sometimes recommended that the client be deprived of sleep on the night before the EEG. Coffee, tea, chocolate, and cola drinks are omitted in the meal before the test because of their stimulating effect. However, meals are not omitted, because an altered blood glucose concentration can cause changes in brain wave patterns. The client is informed that a standard EEG takes 45 to 60 minutes; a sleep EEG requires 12 hours.

A client has been prescribed eye drops for the treatment of glaucoma. At the yearly follow-up appointment, the client tells the nurse that she has stopped using the medication because her vision did not improve. Which action by the nurse is appropriate? Correct response: Explain the therapeutic effect and expected outcome of the medication. Explanation: The nurse needs to explain the therapeutic effect and expected outcome of the medication. The medication is not a cure for glaucoma, but can slow the progression. The client will not see improvements in vision with the use of the medication but should experience little to no deterioration of vision. The doctor may choose to switch the medication, but not because the vision is not improving; it would be based on not obtaining the set intraocular pressure. Administering the medication immediately or referring the client to the emergency department is not appropriate because this is not an emergent situation.

A client presents to the ED reporting a chemical burn to both eyes. Which is the priority nursing intervention? Correct response: Irrigate both eyes. Explanation: The eyes should immediately be irrigated to remove the chemical and preserve the eye. If the chemical is allowed to remain on the eye surface, it may cause ulcerations and permanent damage to the eye. It is appropriate to obtain the MSDS and assess the pH of the corneal surface after irrigation has begun. Irrigation should continue until the pH normalizes. Visual acuity can be assessed once the emergent phase is over.

A nurse is assisting with the clinical examination for determination of brain death for a client, related to potential organ donation. All 50 states in the United States recognize uniform criteria for brain death. The nurse is aware that the three cardinal signs of brain death on clinical examination are all of the following except: Correct response: Glasgow Coma Scale of 6 Explanation: The three cardinal signs of brain death on clinical examination are coma, absence of brain stem reflexes, and apnea. The Glasgow Coma Scale is a tool for determining the client's level of consciousness. A score of 3 indicates a deep coma, and a score of 15 is normal.

A client with spinal trauma tells the nurse she cannot cough. What nursing intervention should the nurse perform when a client with spinal trauma may not be able to cough? Correct response: Suction the airway. Explanation: Suctioning the airway helps remove secretions. An artificial airway increases the production of respiratory secretions. To prevent hypoxemia, the client may need more oxygen than is available in the room air. An endotracheal tube provides an airway from the nose or mouth to an area above the mainstem bronchi. Mechanical ventilation provides a means to regulate the respiratory rate, volume of air, and percentage of oxygen when a client fails to breathe independently.

The nurse is caring for a client after lumbar puncture. The client reports a severe headache. Which actions should the nurse complete? Select all that apply.

Correct response: Maintain the client on bed rest. Administer fluids to the client. Administer analgesic medication. Explanation: When the client assumes an upright position, tension and stretching of the venous sinuses and pain-sensitive structures occur. A postpuncture headache is usually managed by bed rest, analgesic agents, and hydration. Postlumbar puncture headache may be avoided if a small-gauge needle is used and if the client remains prone after the procedure. When more than 20 mL of cerebrospinal fluid is removed, the client is positioned supine for 6 hours.

The nurse is caring for a client with traumatic brain injury (TBI). Which clinical finding, observed during the reassessment of the client, causes the nurse the most concern? Correct response: Temperature increase from 98.0°F to 99.6°F Explanation: Fever in the client with a TBI can be the result of damage to the hypothalamus, cerebral irritation from hemorrhage, or infection. The nurse monitors the client's temperature every 2 to 4 hours. If the temperature increases, efforts are made to identify the cause and to control it using acetaminophen and cooling blankets to maintain normothermia. The other clinical findings are within normal limits.

The nurse in the neurologic ICU is caring for a client who sustained a severe brain injury. Which nursing measures will the nurse implement to help control intracranial pressure (ICP)? Correct response: Maintain cerebral perfusion pressure from 50 to 70 mm Hg Explanation: The nurse should maintain cerebral perfusion pressure from 50 to 70 mm Hg to help control increased ICP. Other measures include elevating the head of the bed as prescribed, maintaining the client's head and neck in neutral alignment (no twisting or flexing the neck), initiating measures to prevent the Valsalva maneuver (e.g., stool softeners), maintaining body temperature within normal limits, administering O2 to maintain PaO2 greater than 90 mm Hg, maintaining fluid balance with normal saline solution, avoiding noxious stimuli (e.g., excessive suctioning, painful procedures), and administering sedation to reduce agitation.

A client seeks care for lower back pain of 2 weeks' duration. Which assessment finding suggests a herniated intervertebral disk? Correct response: Pain radiating down the posterior thigh Explanation: A herniated intervertebral disk may compress the spinal nerve roots, causing sciatic nerve inflammation that results in pain radiating down the leg. Slight knee flexion should relieve, not precipitate, lower back pain. If nerve root compression remains untreated, weakness or paralysis of the innervated muscle group may result; lower leg atrophy may occur if muscles aren't used. Homans' sign is more typical of phlebothrombosis.

What is the most common type of brain neoplasm? Correct response: Glioma Explanation: Gliomas are the most common brain neoplasms, accounting for about 45% of all brain tumors. Angiomas account for approximately 4% of brain tumors. Meningiomas account for 15% to 20% of all brain tumors. Neuromas account for 7% of all brain tumors.

A nurse is working on a surgical floor. The nurse must logroll a client following a: Correct response: laminectomy. Explanation: The client who has had spinal surgery, such as laminectomy, must be logrolled to keep the spinal column straight when turning. The client who has had a thoracotomy or cystectomy may turn himself or may be assisted into a comfortable position. Under normal circumstances, hemorrhoidectomy is an outpatient procedure, and the client may resume normal activities immediately after surgery.

Which statement(s) reflect nursing interventions for a client with post-polio syndrome? Correct response: The nurse provides care aimed at slowing the loss of strength and maintaining the physical, psychological and social well being of the client Explanation: No specific medical or surgical treatment is available for this syndrome and therefore nursing plays a pivotal role in the team approach to assisting clients and families in dealing with the symptoms of progressive loss of muscle strength and significant fatigue. Nursing interventions are aimed at slowing the loss of strength and maintaining the physical, psychological and social well being of the client. Clientss need to plan and coordinate activities to conserve energy and reduce fatigue. Important activities should be planned for the morning as fatigue often increases in the afternoon and evening. Pain in muscles and joints may be a problem. Nonpharmacologic techniques such as the application of heat and cold are most appropriate because these clients tend to have strong reactions to medications.

Which of the following types of hematoma results from venous bleeding with blood gradually accumulating in the space below the dura? Correct response: Subdural Explanation: A subdural hematoma results from venous bleeding, with blood gradually accumulating in the space below the dura. An epidural hematoma stems from arterial bleeding, usually from the middle meningeal artery, and blood accumulation above the dura. An intracerebral hematoma is bleeding within the brain that results from an open or closed head injury or from a cerebrovascular condition such as a ruptured cerebral aneurysm. A cerebral hematoma is bleeding within the skull.

A 24-year-old female rock climber is brought to the emergency department after a fall from the face of a rock. The young lady is admitted for observation after being diagnosed with a contusion to the brain. The client asks the nurse what having a contusion means. How should the nurse respond? Correct response: Contusions are bruising, and sometimes, hemorrhage of superficial cerebral tissue. Explanation: Contusions result in bruising, and sometimes, hemorrhage of superficial cerebral tissue. When the head is struck directly, the injury to the brain is called a coup injury. Dual bruising can result if the force is strong enough to send the brain ricocheting to the opposite side of the skull, which is called a contrecoup injury. Edema develops at the site of or in areas opposite to the injury. A skull fracture can accompany a contusion. Therefore options B, C, and D are incorrect.

A client with a brain tumor experiences projectile vomiting. The nurse integrates understanding of this occurrence as resulting from which of the following? Correct response: Irritation of the meduallary vagal centers Explanation: Vomiting associated with a brain tumor is usually the result of irritation of the vagal centers in the medulla. Edema secondary to the tumor or distortion of the pain-sensitive structures is thought to be the cause of the headache associated with brain tumors. Compression of the surrounding structures results in the signs and symptoms of increased intracranial pressure.

A client comes to the clinic for evaluation because of complaints of dizzinesss and difficulty walking. Further assessment reveals a staggering gait, marked muscle incoordination, and nystagmus. A brain tumor is suspected. Based on the client's assessment findings, the nurse would suspect that the tumor is located in which area of the brain? Correct response: Cerebellum Explanation: Findings such as ataxic or staggering gait, dizziness, marked muscle incoordination, and nystagmus suggest a cerebellar tumor. A frontal lobe tumor frequently produces personality, emotional, and behavioral changes. A tumor in the motor cortex produces seizurelike movements localized on one side of the body. Occipital lobe tumors produce visual manifestations.

Which nursing intervention can prevent a client from experiencing autonomic dysreflexia? Correct response: Monitoring the patency of an indwelling urinary catheter Explanation: A full bladder can precipitate autonomic dysreflexia, the nurse should monitor the patency of an indwelling urinary catheter to prevent its occlusion, which could result in a full bladder. Administering zolpidem tartrate, assessing laboratory values, and placing the client in Trendelenburg's position can't prevent autonomic dysreflexia

A client is admitted to the hospital after sustaining a closed head injury in a skiing accident. The physician ordered neurologic assessments to be performed every 2 hours. The client's neurologic assessments have been unchanged since admission, and the client is complaining of a headache. Which intervention by the nurse is best? Correct response: Assess the client's neurologic status for subtle changes, administer acetaminophen, and then reassess the client in 30 minutes. Explanation: Headache is common after a head injury. Therefore, the nurse should administer acetaminophen to try to manage the client's pain without causing sedation. The nurse should then reassess the client in 30 minutes to note the effectiveness of the pain medication. Administering codeine, an opioid, could cause sedation that may mask changes in the client's neurologic status. Although a headache is expected, the client should receive treatment to alleviate pain. The nurse should notify the physician if the client's neurologic status changes or if treatment doesn't relieve the headache.

A client was undergoing conservative treatment for a herniated nucleus pulposus, at L5 - S1, which was diagnosed by magnetic resonance imaging. Because of increasing neurologic symptoms, the client undergoes lumbar laminectomy. The nurse should take which step during the immediate postoperative period? Correct response: Logroll the client from side to side. Explanation: Logrolling the client maintains alignment of his hips and shoulders and eliminates twisting in his operative area. The nurse should encourage ROM exercises to maintain muscle strength. Because of pressure on the operative area, having the client sit up in a chair or with the head of the bed elevated should be allowed only for short durations.

A client is suspected of having amyotrophic lateral sclerosis (ALS). To help confirm this disorder, the nurse prepares the client for various diagnostic tests. The nurse expects the physician to order: Correct response: electromyography (EMG). Explanation: To help confirm ALS, the physician typically orders EMG, which detects abnormal electrical activity of the involved muscles. To help establish the diagnosis of ALS, EMG must show widespread anterior horn cell dysfunction with fibrillations, positive waves, fasciculations, and chronic changes in the potentials of neurogenic motor units in multiple nerve root distribution in at least three limbs and the paraspinal muscles. Normal sensory responses must accompany these findings. Doppler scanning, Doppler ultrasonography, and quantitative spectral phonoangiography are used to detect vascular disorders, not muscular or neuromuscular abnormalities.

A client with a cerebellar brain tumor is admitted to an acute care facility. The nurse formulates a nursing diagnosis of Risk for injury. Which "related-to" phrase should the nurse add to complete the nursing diagnosis statement? Correct response: Related to impaired balance Explanation: A client with a cerebellar brain tumor may suffer injury from impaired balance as well as disturbed gait and incoordination. Visual field deficits, difficulty swallowing, and psychomotor seizures may result from dysfunction of the pituitary gland, pons, occipital lobe, parietal lobe, or temporal lobe — not from a cerebellar brain tumor. Difficulty swallowing suggests medullary dysfunction. Psychomotor seizures suggest temporal lobe dysfunction.

A client who has just been diagnosed with mixed muscular dystrophy asks the nurse about the usual course of this disease. How should the nurse respond? Correct response: "You may experience progressive deterioration in all voluntary muscles." Explanation: The nurse should tell the client that muscular dystrophy causes progressive, symmetrical wasting of skeletal muscles, without neural or sensory defects. The mixed form of the disease typically strikes between ages 30 and 50 and progresses rapidly, causing deterioration of all voluntary muscles. Because the client asked the nurse this question directly, the nurse should answer and not simply refer the client to the physician. Limb-girdle muscular dystrophy causes a gradual decrease in arm and pelvic muscle strength, resulting in slight disability. Facioscapulohumeral muscular dystrophy is a slowly progressive, relatively benign form of muscular dystrophy; it usually arises before age 10.

A 55-year-old female client presents at the walk-in clinic complaining of feeling like a mask is on her face. While doing the initial assessment, the nurse notes the demonstration of a pill-rolling movement in the right hand and a stooped posture. Physical examination shows bradykinesia and a shuffling gait. What would the nurse suspect is the causative factor for these symptoms? Correct response: Parkinson's disease Explanation:Early signs include stiffness, referred to as rigidity, and tremors of one or both hands, described as pill-rolling (a rhythmic motion of the thumb against the fingers). The hand tremor is obvious at rest and typically decreases when movement is voluntary, such as picking up an object.Bradykinesia, slowness in performing spontaneous movements, develops. Clients have a masklike expression, stooped posture, hypophonia (low volume of speech), and difficulty swallowing saliva. Weight loss occurs. A shuffling gait is apparent, and the client has difficulty turning or redirecting forward motion. Arms are rigid while walking. These symptoms are not indicative of MS, Myesthenia gravis, or Huntington's.

A client with cerebral metastasis suddenly experiences a seizure for which phenytoin 10 mg/kg intravenously is ordered as an initial loading dose. The client weighs 165 pounds. How many milligrams of phenytoin should the client receive? Enter the number ONLY. Correct response:750 Explanation: First, change the client's weight in pounds to kilograms by dividing the weight by 2.2 (2.2 pounds = 1 kg). The client's weight is 75 kg. Next, set up a proportion: 10/1 = x/75; cross multiply and solve for x, which is 750.

A client in the surgical intensive care unit has skeletal tongs in place to stabilize a cervical fracture. Protocol dictates that pin care should be performed each shift. When providing pin care for the client, which finding should the nurse report to the physician? Correct response: A small amount of yellow drainage at the left pin insertion site Explanation: The nurse should report the presence of yellow drainage, which indicates the presence of infection, at the left pin insertion site. Crust formation around the pin site is a natural response to the trauma caused by the pin insertion. Redness at the insertion site may be an early sign of infection; the nurse should continue to monitor the area, but this finding doesn't need to be reported to the physician. The client may experience pain at the pin insertion sites; therefore, the nurse should administer pain medications as ordered. It's necessary to notify the physician only if the pain medication is ineffective.

A client with quadriplegia is in spinal shock. What finding should the nurse expect? Correct response: Absence of reflexes along with flaccid extremities Explanation: During the period immediately following a spinal cord injury, spinal shock occurs. In spinal shock, all reflexes are absent and the extremities are flaccid. When spinal shock subsides, the client will demonstrate positive Babinski's reflex, hyperreflexia, and spasticity of all four extremities.

When caring for a client who is post-intracranial surgery what is the most important parameter to monitor? Correct response: Body temperature Explanation: It is important to monitor the client's body temperature closely because hyperthermia increases brain metabolism, increasing the potential for brain damage. Therefore, elevated temperature must be relieved with an antipyretic and other measures. Options A, B, and C are not the most important parameters to monitor.

A nurse is reviewing a CT scan of the brain, which states that the client has arterial bleeding with blood accumulation above the dura. Which of the following facts of the disease progression is essential to guide the nursing management of client care? Correct response: Monitoring is needed as rapid neurologic deterioration may occur. Explanation: The nurse identifies that the CT scan suggests an epidural hematoma. A key component in planning care is the understanding that rapid neurologic deterioration occurs. Symptoms evolve quickly. A crash cart may be kept nearby, but this is not the key information. An intracerebral hematoma is bleeding within the brain, which is a different area of bleeding.

A patient was admitted to a rehabilitation unit for treatment of a spinal cord injury. The admitting diagnosis is central cord syndrome. During an admissions physical, the nurse expects to find: Correct response: loss of motor power and sensation in the upper extremities. Explanation: Characteristics of a central cord injury include motor deficits (in the upper extremities compared to the lower extremities; sensory loss varies but is more pronounced in the upper extremities); bowel/bladder dysfunction is variable, or function may be completely preserved.

CHP 63 When assessing the pressure of the anterior chamber of the eye, a nurse normally expects to find a pressure of: Correct response: 10 to 20 mm Hg. Explanation: Normally, pressure in the anterior chamber of the eye remains relatively constant at 10 to 20 mm Hg.

The nurse is caring for a comatose client. The nurse knows she should assess the client's motor response. Which method may the nurse use to assess the motor response?

Correct response: Observing the client's response to painful stimulus Explanation: The nurse evaluates motor response in a comatose or unconscious client by administering a painful stimulus. This action helps determine if the client makes an appropriate response by reaching toward or withdrawing from the stimulus. The Romberg test is used to assess equilibrium in a noncomatose client. Pupils are examined for their reaction to light to assess sensitivity in the third cranial (oculomotor) nerve. Sensitivity to temperature, touch, and pain is a test to assess the sensory function of the client and not motor response.

Which intervention is appropriate for a nurse caring for a client in severe pain receiving a continuous I.V. infusion of morphine?

Correct response: Obtaining baseline vital signs before administering the first dose Explanation: The nurse should obtain the client's baseline blood pressure and pulse and respiratory rates before administering the initial dose and then continue to monitor vital signs throughout therapy. A naloxone challenge test may be administered before using an opioid antagonist, not an opioid agonist. The nurse shouldn't discontinue an opioid agonist abruptly because withdrawal symptoms may occur. Morphine commonly is used as a continuous infusion in clients with severe pain regardless of the ability to tolerate fluids.

A geriatric nurse practitioner is assessing older adults. The nurse practitioner knows that older adults sometimes have difficulty following directions during a neurologic examination or diagnostic procedure. What strategies can the nurse practitioner use to examine older clients?

Correct response: Provide brief instructions, one step at a time Explanation: Older adults who have difficulty following directions during a neurologic examination or diagnostic procedure need brief instructions given one step at a time during the examination or procedure. In addition, diseases that are more common in older adults, such as dementia, often make it difficult to perform a neurologic assessment. The nurse should not offer incentives to them. In addition, spreading the examination over a couple of days or suggesting an examiner of their age may not help in examining older adults.

The nurse who is employed in a neurologist's office is performing a history and assessment on a client experiencing hearing difficulty. The nurse is most correct to gather equipment to assess the function of cranial nerve:

Correct response: VIII Explanation: There are 12 pairs of cranial nerves. Cranial nerve VIII is the vestibulocochlear or auditory nerve responsible for hearing and balance. Cranial nerve II is the optic nerve. Cranial nerve VI is the abducens nerve responsible for eye movement. Cranial nerve XI is the accessory nerve and is involved with head and shoulder movement.

The nurse is performing a neurologic assessment on a client diagnosed with a stroke and cannot elicit a gag reflex. This deficit is related to which of the following cranial nerves?

Correct response: X Explanation: CN X is the vagus nerve and has to do with the gag reflex, laryngeal hoarseness, swallowing ability, and the symmetrical rise of the uvula and soft palate. CN VII is the facial nerve and has to do with symmetry of facial movements and the ability to discriminate between the tastes of sugar and salt. The inability to close one eyelid indicates impairment of this nerve. CN VIII is the acoustic nerve. It has to do with hearing, air and bone conduction, and balance. CN III is the oculomotor nerve and has to do with pupillary response, conjugate movements, and nystagmus.

The nurse is caring for a client in the emergency department with a diagnosis of head trauma secondary to a motorcycle accident. The nurse aide is assigned to clean the client's face and torso. The nurse would provide further instruction after seeing that the nurse aide:

Correct response: moved the client's head to clean behind the ears. Explanation: Further instruction would be provided to the nurse aide when the nurse aide attempted to move the client's head to clean behind the ears. There should be no movement of the client's head when there is a history of head trauma. Cleaning the client's face with soapy water, cleaning the eye area, and cleaning the neck and upper chest are all appropriate actions completed by the nurse aide.

The nurse has completed evaluating the client's cranial nerves. The nurse documents impairment of the right cervical nerves (CN IX and CN X). Based on these findings, the nurse should instruct the client to

Correct response: refrain from eating or drinking for now. Explanation: Significant findings of CN IX (glossopharyngeal) include difficulty swallowing (dysphagia) and impaired taste, and significant findings of CN X (vagus) include weak or absent gag reflex, difficulty swallowing, aspiration, hoarseness, and slurred speech (dysarthria). Based on these findings, the nurse should instruct the client to refrain from eating and drinking and should contact the health care provider. The other instructions are associated with abnormalities of CN II (optic) and CN VIII (acoustic).

A nurse is caring for a client with an injury to the central nervous system. When caring for a client with a spinal cord insult slowing transmission of the motor neurons, the nurse would anticipate a delayed reaction in:

Correct response: response due to interrupted impulses from the central nervous system Explanation: The central nervous system is composed of the brain and the spinal cord. Motor neurons transmit impulses from the central nervous system. Slowing transmission in this area would slow the response of transmission leading to a delay in reaction. Sensory neurons transmit impulses from the environment to the central nervous system, allowing identification of a stimulus. Cognitive centers of the brain interpret the information.

A nurse and nursing student are caring for a client recovering from a lumbar puncture yesterday. The client reports a headache despite being on bedrest overnight. The physician plans an epidural blood patch this morning. The student asks how this will help the headache. The correct reply from the nurse is which of the following?

Correct response:"The blood will seal the hole in the dura and prevent further loss of cerebral spinal fluid." Explanation: Loss of CSF causes the headache. Occasionally, if the headache persists, the epidural blood patch technique may be used. Blood is withdrawn from the antecubital vein and injected into the site of the previous puncture. The rationale is that the blood will act as a plug to seal the hole in the dura and preven further loss of CSF. The blood is not put into the subarachnoid space. The needle is inserted below the level of the spinal cord, which prevents damage to the cord. It is not a lack of moisture that prevents healing; it is more related to the size of the needle used for the puncture.

The nurse is performing a neurological assessment of a client who has sustained damage to the frontal cortex. Which of the following deficits will the nurse look for during assessment?

Correct response:The inability to tell how a mouse and a cat are alike Explanation: The client with damage to the fronal cortex will display a deficit in intellectual functioning. Questions designed to assess this capacity might include the ability to recognize similarities: for example, how are a mouse and dog or pen and pencil alike? The Romberg test assesses balance, which has to do with the cerebellar and basal ganglia influence on the motor system. Absence of movement below the waist suggests a deficit with the spinal cord. Intentional tremors have to do with deficits of the motor system.

While stopped at a stop sign, a patient's car was struck from behind by another vehicle. The patient sustained a cerebral contusion and was admitted to the hospital. During what time period after the injury will the effects of injury peak? Correct response: 18 to 36 hours Explanation: Contusions are characterized by loss of consciousness associated with stupor and confusion. Other characteristics can include tissue alteration and neurologic deficit without hematoma formation, alteration in consciousness without localizing signs, and hemorrhage into the tissue that varies in size and is surrounded by edema. The effects of injury (hemorrhage and edema) peak after about 18 to 36 hours.

For a patient with an SCI, why is it beneficial to administer oxygen to maintain a high partial pressure of oxygen (PaO2)? Correct response: Because hypoxemia can create or worsen a neurologic deficit of the spinal cord Explanation: Oxygen is administered to maintain a high partial pressure of arterial oxygen (PaO2) because hypoxemia can create or worsen a neurologic deficit of the spinal cord.

A patient with a brain tumor is complaining of headaches that are worse in the morning. What does the nurse know could be the reason for the morning headaches? Correct response: Increased intracranial pressure Explanation: Headache, although not always present, is most common in the early morning and is made worse by coughing, straining, or sudden movement. It is thought to be caused by the tumor invading, compressing, or distorting the pain-sensitive structures or by edema that accompanies the tumor, leading to increased intracranial pressure.

The daughter of a patient with Huntington's disease asks the nurse what the risk is of her inheriting the disease. What is the best response by the nurse? Correct response: "If one parent has the disorder, there is a 50% chance that you will inherit the disease." Explanation: Huntington disease is a chronic, progressive, hereditary disease of the nervous system that results in progressive involuntary choreiform movement and dementia. The disease affects approximately 1 in 10,000 men or women of all races at midlife. It is transmitted as an autosomal dominant genetic disorder; therefore, each child of a parent with Huntington disease has a 50% risk of inheriting the disorder (Ha & Fung, 2012).

The nurse is providing end-of-life care to a client who was diagnosed with glioblastoma multiforme (GBM) 8 months ago. Despite a calm interaction with the client 1 hour ago, the client is now angry and yells, "Get out of my room and don't touch me anymore. I don't need your help!" How should the nurse respond? Correct response: "I can tell now is not the right time for me to come in and check on you. Please let me know when it is a better time for me to come back." Explanation: Personality changes, mood swings and irritability can be common manifestations of both growth of the brain tumor and also the process of grief and loss, such as in the case of the client who is receiving end-of-life care. The client's anger and yelling at the nurse is indicative of ineffective coping and warrants the nurse to take a therapeutic approach when responding to the anger. Acknowledging that the client is not ready to receive care at the moment and asking the client to contact the nurse when he or she is ready enables to client to maintain control and promotes self-esteem. Telling the client to speak to the nurse's supervisor does not promote a strong nurse-patient relationship and is not a supportive way to manage end-of-life care. Telling the client that he or she is not permitted to speak to the nurse "that way" may increase the client's anger and puts limits on the client's sense of control. This response does not promote an effective nurse-patient relationship. The nurse must use extra caution when responding to a client who is experiencing emotional swings when faced with death and dying. By stating, "I can see you no longer want me as your nurse," the nurse is making an assumption that the client does not want him or her as the nurse any longer. By making this statement, the nurse is limiting opportunities for the client to verbalize feelings and emotions related to stress, grief and loss.

The nurse educator is teaching nursing students about various types of brain tumors. The instructor recognizes that teaching has been effective when students correctly identify a client whose lab work indicates excessively high levels of thyroid stimulating hormone would most likely be diagnosed with which type of tumor? Correct response: Pituitary adenoma Explanation: Pituitary adenomas can increased production of several hormones including TSH, ACTH, growth hormone and prolactin. Excessive hormone production is not characteristic of the brain tumors identified in the alternate options.

A 37-year-old mother of three has just been diagnosed with a grade I meningioma. As part of patient education, the nurse tells the patient that: Correct response: Growth is slow and symptoms are caused by compression rather than tissue invasion. Explanation: A meningioma is benign, encapsulated, and slow-growing. Sometimes the patient has no symptoms because of the slow-growing nature of the tumor.

The nurse is caring for a client who has been hospitalized for investigation of a sudden change in gait due to loss of balance and coordination. A magnetic resonance imaging scan reveals the client has a brain tumor. On or close to which brain structure is the tumor most likely situated? Correct response: Cerebellum Explanation: The cerebellum is the brain structure responsible for balance, coordination and fine muscle control. The tumor is most likely located on or near this brain structure. A tumor located on or near the brain stem would more likely cause changes in autonomic functioning such as blood pressure. The temporal lobe is responsible for language comprehension, behavior, memory, hearing and emotions. A tumor effecting the pituitary gland would result in hormonal changes as this structure is responsible for hormones, growth and reproductive processes in the body.

The nurse is seeing the mother of a client who states, "I'm so relieved because my son's doctor told me his brain tumor is benign." The nurse knows what is true about benign brain tumors? Correct response: They can affect vital functioning. Explanation: Benign tumors are usually slow growing but can occur in a vital area, where they can grow large enough to cause serious effects. Surgical removal of a benign tumor is dependent on many factors; even if the tumor is slow growing or not growing at all, the location of the tumor in the brain factors into the decision for surgical removal. The prognosis for all brain tumors is not necessarily poor. Treatment is individualized and can have varying prognostic outcomes. Benign tumors are not metastatic, meaning they do not grow rapidly or spread into surrounding tissue, but they can still be considered life-threatening.

The nurse is caring for a client who underwent surgery to remove a spinal cord tumor. When conducting the postoperative assessment, the nurse notes the presence of a bulge at the surgical site. The nurse suspects the client is experiencing what complication from the surgery? Correct response: Cerebrospinal fluid leakage Explanation:Bulging at the incision may indicate a contained cerebrospinal fluid (CSF) leak. The site should be monitored for increasing bulging, known as pseudomeningocele, which may require surgical repair. Infection at the surgical site should be suspected if the surgical dressing is stained. The bulge does not indicate growth of secondary tumor, this can only be identified using diagnostic imaging. Impaired tissue healing would be indicated if the nurse assessed redness, swelling and warmth at the surgical site during a dressing change. The bulge at the site warrants further assessment of a postsurgical leak of CSF.

A client with post-polio syndrome displays fatigue and decreased muscle strength. How should the nurse best respond to the client? Correct response: "Intravenous immunoglobulin infusion may help you." Explanation: There is no specific treatment for post-polio syndrome; however, the infusion of IV immunoglobulin has been shown to help with the physical pain and weakness. Sleeping and relaxation may not assist the client with post-polio syndrome. The syndrome is very common and is most likely related to the past diagnosis of polio.

The nurse is caring for a patient with Huntington's disease in the long-term care facility. What does the nurse recognize as the most prominent symptom of the disease that the patient exhibits? Correct response: Rapid, jerky, involuntary movements Explanation: The most prominent clinical features of the disease are chorea (rapid, jerky, involuntary, purposeless movements), impaired voluntary movement, intellectual decline, and often personality changes (Aubeeluck & Wilson, 2008).

A nurse is assessing pain in a client who has a spinal cord injury. The client states that even a light touch to the legs will illicit severe pain. The client is describing which type of pain? Correct response: allodynia Explanation: Allodynia is a type of neurogenic pain whereby clients experience pain in response to a normally painless stimulus. Hyperalgesia is a type of neurogenic pain whereby clients experience an increased response to a painful stimulus. Nociceptive pain is detected by specialized sensory nerves located throughout the soft tissues and is not neurogenic. Idiopathic pain has no apparent underlying cause and is not neurogenic.

The nurse is caring for a postoperative client who had surgery to decrease intracranial pressure after suffering a head injury. Which assessment finding is promptly reported to the physician? Correct response: The client's vital signs are temperature, 100.9° F; heart rate, 88 beats/minute; respiratory rate, 18 breaths/minute; and blood pressure, 138/80 mm Hg. Explanation: The assessment finding promptly reported to the physician is the information which may cause complications. It is important to report the elevation in client temperature (100.9° F) because hyperthermia increases brain metabolism, increasing the potential for brain damage. It is not unusual for the client to experience periorbital edema and ecchymosis secondary to the head injury and surgery. Improved level of consciousness is a positive outcome of the treatment provided. There is no complication related to semi-Fowler's position.

A client has undergone a cervical discectomy. The nurse determines which interventions are essential to teach the client? Select all that apply. Correct response: Keep staples or sutures clean and dry Cover incision with dry dressing Call health care provider if the area is red or irritated Avoid twisting or flexing the neck Do not remove dressing until the next visit Explanation: The client needs to keep staples or sutures clean and dry and covered with a dry dressing. Symptoms of infection should be reported to the health care provider. Twisting and flexing of the neck should be avoided. The client can remove the dressing to change it. The client should not sit or stand for longer than half an hour at a time.

The nurse is performing an assessment for a patient in the clinic with Parkinson's disease. The nurse determines that the patient's voice has changed since the last visit and is now more difficult to understand. How should the nurse document this finding? Correct response: Dysphonia Explanation:Dysphonia (voice impairment or altered voice production) may occur as a result of weakness and incoordination of the muscles responsible for speech.

Which statement indicates appropriate nursing intervention for a client with post-polio syndrome? Correct response: Provide care aimed at slowing the loss of strength and maintaining overall well-being. Explanation: No specific medical or surgical treatment is available for this syndrome and therefore nursing plays a pivotal role in the team approach to assisting clients and families in dealing with the symptoms of progressive loss of muscle strength and significant fatigue. Nursing interventions are aimed at slowing the loss of strength and maintaining the physical, psychological and social well-being of the client. Clients need to plan and coordinate activities to conserve energy and reduce fatigue. Important activities should be planned for the morning as fatigue often increases in the afternoon and evening. Pain in muscles and joints may be a problem. Nonpharmacologic techniques, such as the application of heat and cold, are most appropriate because these clients tend to have strong reactions to medications.

The nurse reviews the patient's drug regimen for treatment of a brain tumor. She explains to the patient why one of the following drugs would not be prescribed, even though it might have therapeutic benefits. Which drug would not be prescribed for this patient? Correct response: Coumadin Explanation: Although deep vein thrombosis and pulmonary embolism occur in about 15% of patients and cause significant morbidity, anticoagulants are not prescribed due to the risk for CNS hemorrhage.

A client with a T4-level spinal cord injury (SCI) is experiencing autonomic dysreflexia; his blood pressure is 230/110. The nurse cannot locate the cause and administers antihypertensive medication as ordered. The nurse empties the client's bladder and the symptoms abate. Now, what must the nurse watch for? Correct response: Rebound hypotension Explanation: When the cause is removed and the symptoms abate, the blood pressure goes down. The antihyperstensive medication is still working. The nurse must watch for rebound hypotension. Rebound hypertension is not an issue. Spinal shock occurs right after the initial injury. The client is not at any more risk for a urinary tract infection after the episode than he was before.

Which term refers to the shifting of brain tissue from an area of high pressure to an area of low pressure? Correct response: Herniation Explanation: Herniation refers to the shifting of brain tissue from an area of high pressure to an area of lower pressure. Autoregulation is an ability of cerebral blood vessels to dilate or constrict to maintain stable cerebral blood flow despite changes in systemic arterial blood pressure. Cushing's response is the brain's attempt to restore blood flow by increasing arterial pressure to overcome the increased ICP. The Monro-Kellie hypothesis is a theory that states that, due to limited space for expansion within the skull, an increase in any one of the cranial contents causes a change in the volume of the others.

A client with weakness and tingling in both legs is admitted to the medical-surgical unit with a tentative diagnosis of Guillain-Barré syndrome. On admission, which assessment is most important for this client? Correct response: Lung auscultation and measurement of vital capacity and tidal volume Explanation: In Guillain-Barré syndrome, polyneuritis commonly causes weakness and paralysis, which may ascend to the trunk and involve the respiratory muscles. Lung auscultation and measurement of vital capacity, tidal volume, and negative inspiratory force are crucial in detecting and preventing respiratory failure — the most serious complication of polyneuritis. A peripheral nerve disorder, polyneuritis doesn't cause increased ICP. Although the nurse must evaluate the client for pain and discomfort and must assess the nutritional status and metabolic state, these aren't priorities.

Which type of brain injury has occurred if the client can be aroused with effort but soon slips back into unconsciousness? Correct response: Contusion Explanation: Contusions can be characterized by loss of consciousness associated with stupor and confusion. A concussion is a temporary loss of neurologic function with no apparent structural damage. A diffuse axonal injury involves widespread damage to the axons in the cerebral hemispheres, corpus callosum, and brainstem. An intracranial hemorrhage is a collection of blood that develops within the cranial vault.

The nurse working on the neurological unit is caring for a client with a basilar skull fracture. During assessment, the nurse expects to observe Battle's sign, which is a sign of basilar skill fracture. Which of the following correctly decribes Battle's sign? Correct response: Ecchymosis over the mastoid Explanation: With fractures of the base of the skull, an area of ecchymosis (bruising) may be seen over the mastoid and is called Battle's sign. Basilar skull fractures are suspected when cerebrospinal fluid escapes from the ears or the nose.

You are a neurotrauma nurse working in a neuro ICU. What would you know is an acute emergency and is seen in clients with a cervical or high thoracic spinal cord injury after the spinal shock subsides? Correct response: Autonomic dysreflexia Explanation: Autonomic dysreflexia is an acute emergency and is seen in clients with a cervical or high thoracic spinal cord injury, usually after the spinal shock subsides. Tetraplegia results in the paralysis of all extremities when there is a high cervical spine injury. Paraplegia occurs with injuries at the thoracic level. Areflexia is a loss of sympathetic reflex activity below the level of injury within 30 to 60 minutes of a spinal injury.

A patient was body surfing in the ocean and sustained a cervical spinal cord fracture. A halo traction device was applied. How does the patient benefit from the application of the halo device? Correct response: It allows for stabilization of the cervical spine along with early ambulation. Explanation: Halo devices provide immobilization of the cervical spine while allowing early ambulation.

A patient is admitted to the emergency room with a fractured skull sustained in a motorcycle accident. The nurse notes fluid leaking from the patient's ears. The nurse knows this is a probable sign of which type of skull fracture? Correct response: Basilar Explanation: Basilar skull fractures are suspected when cerebrospinal fluid (CSF) escapes from the ears (CSF otorrhea) and/or the nose (CSF rhinorrhea).

A nurse is assessing a pediatric client in a public health clinic. The parent states that the client has been sneezing and rubbing the eyes. The nurses observes the client's eyes and documents objective symptoms of watery and red eyes. When reporting to the physician the assessment findings, which word is appropriate? Correct response: Signs and symptoms of conjunctivitis Explanation: Conjunctivitis often stems from an allergy causing inflammation of the conjunctiva, which is a thin, transparent mucous membrane. Conjunctivitis can cause symptoms of itchiness, redness, and watery eyes. Ptosis is drooping of the upper eyelid. Proptosis is an extended and upper eyelid that delays in closing or remains partially open. Nystagmus is an uncontrolled oscillating movement of the eyeball.

An eight-grade boy tells the school nurse that the eye doctor told him he had astigmatism and that meant his eyeball wasn't shaped right. The boy says he went home and looked in the mirror and both eyes looked just alike. What is the school nurse's best response? Correct response: "Astigmatism means that the cornea of the eye is shaped differently than the cornea in most eyes." Explanation: Astigmatismis visual distortion caused by an irregularly shaped cornea. Many people have both astigmatism and myopia or hyperopia. The other options are incorrect because they are not the best answer.

The nurse is preparing a client for a neurological examination by the physician and explains tests the physician will be doing, including the Romberg test. The client asks the purpose of this particular test. The correct reply by the nurse is which of the following?

Correct response: "It is a test for balance." Explanation: The Romberg test screens for balance. The client stands with feet together and arms at the side, first with eyes open and then with both eyes closed for 20 to 30 seconds. Slight swaying is normal, but a loss of balance is abnormal and is considered a positive Romberg test.

A client who was found unconscious at home is brought to the hospital by a rescue squad. In the intensive care unit, the nurse checks the client's oculocephalic (doll's eye) response by:

Correct response: turning the client's head suddenly while holding the eyelids open. Explanation: To elicit the oculocephalic response, which detects cranial nerve compression, the nurse turns the client's head suddenly while holding the eyelids open. Normally, the eyes move from side to side when the head is turned; in an abnormal response, the eyes remain fixed. The nurse introduces ice water into the external auditory canal when testing the oculovestibular response; normally, the client's eyes deviate to the side of ice water introduction. The nurse touches the client's cornea with a wisp of cotton to elicit the corneal reflex response, which reveals brain stem function; blinking is the normal response. Shining a bright light into the client's pupil helps evaluate brain stem and cranial nerve III function; normally, the client's pupil responds by constricting.

The nurse is caring for a client who was diagnosed with a glioma 5 months ago. Today, the client was brought to the emergency department by his caregiver because he collapsed at home. The nurse suspects late signs of rising intracranial pressure (ICP) when which blood pressure and pulse readings are noted? Correct response: BP = 175/45 mm Hg; HR = 42 bpm Explanation: With a blood pressure of 175/45 mm Hg, it is evident that this client is experiencing progressively rising ICP, resulting from an advanced stage of the brain tumor. This blood pressure demonstrates a wide pulse pressure, meaning the difference between systolic and diastolic pressure is large. A heart rate of 42 bpm indicates the client is bradycardic. This finding paired with hypertensive blood pressure with a widening pulse pressure are part of the Cushing triad related to increased ICP.

The nurse educator is providing orientation to a new group of staff nurses on an oncology unit. Part of the orientation is to help nurses understand the differences between various types of brain tumors. The nurse educator correctly identifies that glioma tumors are classified based on the fact that they originate where in the brain? Correct response: Within the brain tissue Explanation: Gliomas tumors are a type of intracerebral brain neoplasm. They originate within brain tissue. Tumors arising from the coverings of the brain include meningiomas. These tumors grow on the membrane covering of the brain, called the meninges. An acoutstic neuroma is an example of tumors that grow out of or on cranial nerves and cause compression leading to sensory deficits. Metastasis refers to spreading of any kind of malignant primary tumor. This term is not specific to any one classification of tumor.

A male client who has undergone a cervical discectomy is being discharged with a cervical collar. Which of the following would be most appropriate to include the client's discharge plan? Correct response: Keeping the head in a neutral position Explanation: After a cervical discectomy, the client typically wears a cervical collar. The client should be instructed to keep his head in a neutral position and wear the collar at all times unless the physician has instructed otherwise. The front part of the collar is removed for shaving and the neck should be kept still while the collar is open or off.

The nursing instructor gives their students an assignment of making a plan of care for a client with Huntington's disease. What would be important for the students to include in the teaching portion of the care plan? Correct response: How to facilitate tasks such as using both hands to hold a drinking glass Explanation: The nurse demonstrates how to facilitate tasks such as using both hands to hold a drinking glass, using a straw to drink, and wearing slip-on shoes. The teaching portion of the care plan would not include how to exercise, perform household tasks, or take a bath.

A client with a malignant brain tumor comes to the clinic for a follow up. During the visit, the client asks the nurse, "Why am I so tired all the time?" When responding to the client, which information would the nurse include as possible causes? Select all that apply. Correct response: Tumor Treatment being used Stress Explanation: Fatigue is a symptom experienced by clients with both malignant and nonmalignant brain tumors. Etiology of fatigue can be multifactorial. The tumor itself, surgery, medications, chemotherapy, and radiation may all contribute to increased fatigue. Clients may report a constant feeling of exhaustion, weakness, and lack of energy. It is also important to identify underlying conditions, such as stress, anxiety, and depression, which may play a role in fatigue. Metastasis and increased intracranial pressure are not usually associated with fatigue.

The nursing is assessing a client who has been diagnosed with a pituitary adenoma, but has not yet started treatment. The client reports having increased heart rate, hand tremors, difficulty sleeping, weight loss and hyperthermia. The nurse anticipates the client will require blood work to assess for overproduction of which hormone? Correct response: Thyroid-stimulating hormone Explanation: In clients diagnosed with pituitary tumors, increase may be seen in prolactin hormone, growth hormone, adrenocorticotropic hormone, or thyroid-stimulating hormone. In this case, the client is exhibiting symptoms related to hyperthyroidism and the blood work should include the thyroid-stimulating hormone level to determine if an overproduction of this hormone due to the presence of the tumor is the cause of the presenting symptoms.

Which are characteristics of autonomic dysreflexia? Correct response: severe hypertension, slow heart rate, pounding headache, sweating Explanation: Autonomic dysreflexia is an exaggerated sympathetic nervous system response. Hypertension, tachycardia, bradycardia, and flushed skin would occur.

Which finding indicates increasing intracranial pressure (ICP) in the client who has sustained a head injury? Correct response: Widened pulse pressure Explanation: Signs of increasing ICP include slowing of the heart rate (bradycardia), increasing systolic blood pressure, and widening pulse pressure (Cushing reflex). As brain compression increases, respirations become rapid, blood pressure may decrease, and the pulse slows further. This is an ominous development, as is a rapid fluctuation of vital signs. Temperature is maintained at less than 38°C (100.4°F). Tachycardia and arterial hypotension may indicate that bleeding is occurring elsewhere in the body.

A nurse is assisting during a lumbar puncture. How should the nurse position the client for this procedure?

Correct response: Lateral recumbent, with chin resting on flexed knees Explanation: To maximize the space between the vertebrae, the client is placed in a lateral recumbent position with knees flexed toward the chin. The needle is inserted between L4 and L5. The other positions wouldn't allow as much space between L4 and L5.

The client is taking oxycodone (Oxycontin) for chronic back pain and reports decreased pain relief when he began taking a herb to improve his physical stamina. The nurse asks if the herb is

Correct response: ginseng Explanation: Ginseng may inhibit the analgesic effects of an opioid, such as oxycodone. The other herbs listed (valerian, kava-kava, and chamomile) may increase central nervous system depression.

A client is admitted to an acute care facility with a suspected dysfunction of the lower brain stem. The nurse should monitor this client closely for:

Correct response: hypoxia. Explanation: Lower brain stem dysfunction alters bulbar functions, such as breathing, talking, swallowing, and coughing. Therefore, the nurse should monitor the client closely for hypoxia. Temperature control, vision, and gait aren't lower brain stem functions.

Prostaglandins are chemical substances with what property?

Correct response: Increase the sensitivity of pain receptors Explanation: Prostaglandins are believed to increase sensitivity to pain receptors by enhancing the pain-provoking effect of bradykinin. Endorphins and enkephalins reduce or inhibit transmission or perception of pain. Morphine and other opioid medications inhibit the transmission of noxious stimuli by mimicking enkephalin and endorphin.


Set pelajaran terkait

Communication 457 Quiz 3-Scripts, Episodes, and Nonverbal Communication

View Set

Chapter 6 General Anatomy: Fill in the Blanks

View Set

Political Statement: from the Universal Declaration of Human Rights

View Set

Ch. 26, Bipolar and related disorders

View Set

The 8 Elements of Humanistic Psychology

View Set

az-900 sample questions 11-22-22

View Set